Sunteți pe pagina 1din 52

- ISTORIA MATEMATICII

Oricare ar fi durata timpului, tiina


ntrebuinrii lui l va face lung
Seneca


1. Istoria Matematicii

Istoria calendarului
de prof. Adrian Stan

Etimologic, cuvntul calendar vine de la latinescul calendarium ce reprezenta primele zile
din lun i care se numeau calende - zilele n care se plteau datoriile. Calendarul astronomic reprezint
modul de divizare a timpului dup observarea fenomenelor astronomice ciclice cum ar fi fazele Lunii
calendar lunar, sau dup micarea aparent a Soarelui calendar solar.
n Egipt acum circa 2700 ani .e.n a aprut primul calendar solar cu care se
putea calcula cu exactitate anul, avnd 12 luni a cte 30 de zile, n total 360 de zile
la care se aduga la sfritul anului nc cinci zile. Pn la acesta, egiptenii foloseau
din anul 2776 .e.n, un calendar lunar, n funcie de fazele Lunii, ce ncepea o dat
cu anotimpul inundaiilor asociat cu rsritul stelei Sirius la mijlocul lunii iulie.
Este interesant c egiptenii au gsit cu o precizie uimitoare Nordul geografic(cu o
deviaie de cel mult un grad), orientnd feele marilor piramide n direcia celor
patru puncte cardinale.
Civilizaia sumerienilor a realizat gruparea stelelor n constelaii fapt ce a permis ntocmirea unui
calendar lunar si mai apoi unul lunisolar cu 12 luni avnd 354 de zile. Diferena de 11 zile fa de anul solar
se realiza prin adugarea din trei n trei ani a unei luni.
Babilonienii folosind sistemul sexagesimal preluat de la sumerieni au fost primii din lume, acum
circa 2000 de ani .e.n, care au mprit cercul n 360 de grade, gradul n 60 de minute iar minutul n 60 de
secunde. Acestea au fost mai departe preluate de evrei, greci i mai apoi de romani ajungnd pn n vremea
noastr.
De asemenea, ei mpreau i anul n 360 de zile. Din sutele de mii de tblie de lut descoperite,
tiina mesopotamian a excelat n algebr i astronomie, fiind primii care au fcut diferena dintre o stea i o
planet, primii care au determinat solstiiile i echinociile, i de asemenea ei au fost primii care au mprit
ecliptica n dousprezece zodii. Prima hart a boltei cereti avea notate cu exactitate orbitele, conjunciile i
eclipsele principalelor planete. Uimitor este faptul cum au reuit s prevad foarte multe din eclipsele de
lun.
n Mesopotamia acum circa 500 de ani .e.n era folosit un calendar lunar dar la care pe parcursul a
19 ani se adugau apte luni suplimentare pentru a-l putea potrivi cu anul solar.
i n Europa , n zona de sud a Germaniei, la Ezelsdorf, s-a descoperit un corp conic din aur din
perioada Bronzului, acum circa 970 ani .e.n, avnd gravate pe el o serie de cercuri ce simbolizau zile.
Adunate, acestea ddeau 354 respectiv 365 zile indicnd de fapt un calendar lunar-solar.
n mitologia greac, transmis de ctre poetul grec Hesiod, Timpul numit Cronos este fiul rezultat
dintre Uranus Cerul i Geea Pmntul. Lumea zeitilor este ct se poate de uman reprezentat, n
scrierile pstrate ale Antichitii, cci se amintete de Cronos ca cel care i-a ucis tatl i i-a nghiit fraii
din care doar Zeus a scpat.
tiina greac i are n prim plan pe Aristarh din Samos(310-230 .e.n) care n sec. III .e.n n
tratatul Despre dimensiunile i distanele Soarelui i Lunii afirma c Pmntul efectueaz o micare de
rotaie de 24 de ore, n jurul axei sale. De asemenea, amintete faptul c stelele i Soarele sunt fixe iar
Pmntul se nvrte n jurul Soarelui descriind un cerc, gndire ce este premergtoare sistemului heliocentric
al lui Copernic (1473 - 1543)i care din pcate nu este mprtit de mai toi savanii Antichitii. Chiar i
Copernic a trebuit s asiste la interzicerea n 1616 a operelor sale despre sistemul heliocentric de ctre Papa
Paul al V-lea.
Heraclit din Pont (circa 350 ani .e.n) discipol al lui Platon contribuie la formarea unei imagini
astronomice despre Univers. Pentru el, bolta cu stele este fix iar Pmntul se rotete o dat pe zi n jurul
axei sale, micarea aceasta de rotaie fiind sensul pentru succesiunea zi- noapte.
De abia n 1851, fizicianul francez Jean Bernard Focault (1819-1868) cu ajutorul unui pendul

- ISTORIA MATEMATICII -

(pendulul lui Focault o frnghie lung de 67 m i o bil de 28 kg)
prezint publicului dovada tiinific a rotirii Pmntului n jurul propriei
axe, demonstrnd ideea heliocentric a lui Copernic (Soarele n centrul
Universului). Micarea spaiului de sub pendul reprezenta dovada rotirii
Pmntului.
Un mare nvat al
timpului su, Eratostene din
Cirene(circa 240 .e.n) consider
Pmntul ca avnd form sferic i reuete cu o precizie uimitoare
pentru timpul su s deduc lungimea circumferinei Pmntului de
39690 km foarte aproape de valoarea real de 40075 km.
n anul 153 .e.n anul oficial va ncepe la 1 ianuarie i nu la
1 martie ca pn atunci. Lunile septembrie (luna a aptea),
octombrie(luna a opta), noiembrie(luna a noua), decembrie(luna a
zecea) ni s-au pstrat din acea perioad.
Primul calendar cu an bisect este introdus prin decret de ctre regele egiptean Ptolemeu al III-lea ,
pentru a elimina decalajul de o zi la patru ani.
Un alt matematician i astronom grec din sec. II .e.n , Hiparchos , a fost primul care a introdus n
tiina greac mprirea cercului n 360 de grade, gradul n 60 de minute, i minutul n 30 secunde i de
asemenea a construit o serie de instrumente astronomice folosite mult timp i
dup el ce i-au permis s determine anul solar cu o precizie uimitoare, la o
diferen de cteva secunde de cel real i s determine precesia echinociilor-
fenomenul de deplasare anual a punctelor echinoxiale.
n sec. II e.n Ptolemeu (87- 165 e.n ) creatorul sistemului geocentric a
reprezentat vrful tiinei elenistice prin tratatele acestuia despre optic,
astronomie, acustic, teoria planetelor i stabilirea unui calendar al rsritului i
apusului atrilor.
i n Imperiul Roman, Iulius Cesar sftuit de astronomul egiptean
Sosigene, introduce n anul 46 .e.n calendarul ce-i poart numele, calendarul
iulian, i care intr n vigoare la 1 ianuarie 45 .e.n, iar din trei n trei ani i va fi adugat un an bisect. El va fi
corectat n anul 9 .e.n de ctre mpratul Augustus care ncepnd cu anul 8 .e.n i din patru n patru ani este
introdus anul bisect. Denumirea lunii iulie provine de la latinescul Iulius dat n cinstea mpratului roman.
Mai mult, o dat cu expansiunea cretinismului n Imperiul Roman, calendarul ncepe s calculeze timpul de
la naterea lui Isus Cristos i nu de la ntemeierea Romei, din sec. VIII, .e.n.
O nou reform a calendarului care se pstreaz pn astzi este realizat de ctre
papa Grigore al XIII-lea n 1582 pentru a ajusta diferena de zece zile dintre data actual
conform calendarului iulian i cea a anului solar. Astfel, calendarul gregorian numit aa n
cinstea papei, face ca ziua de 4 octombrie s devin 15 octombrie.
n Rusia, nceputul anului se fcea la 1 septembrie, i o dat cu trecerea de la
calendarul bizantin la cel iulian data de 1 septembrie 1700 devine 1 ianuarie 1700.
Primul calendar oficial n China dateaz din anul 104 .e.n i se datoreaz
observaiilor astronomice ce au permis chinezilor s asocieze constelaiilor observate, succesiunea
anotimpurilor. Anul era mprit n 365 de zile i un sfert, attea grade avnd i cercul. Primul mare
observator din lume dotat cu un mecanism de orologerie a fost inventat de chinezi n anul 1088.
Calendarul maya aduce n prim plan divinitatea creia populaiile precolumbiene ncepnd cu sec
5 . e. n i datoreaz cunoaterea scrierii iar nelegerea cosmosului i spiritualitatea nsemna acelai lucru.
Calendarul maya era mprit n 18 perioade (numite uinal) a cte 20 de zile fiecare reprezentate de un
caracter, n total 360 de zile, la care se aduga 5 zile nefaste care nu aveau nume deoarece se considerau
nefaste. Separat aveau i un calendar pentru ritualurile divine i de asemenea un calendar numit marele ciclu
de 5200 tun (1 tun = 360 zile) ce cuprinde toat istoria lor i care pleac de la data de 13 august 3114 i.e.n i
care se ncheie n 21 decembrie 2012 i ar fi nsemnat sfritul lumii dup unii. Interesant este c la mayai
spre deosebire de noi apare anul zero.
Bibliografie: 1. Istoria culturii i civilizaiei. Ovidiu Drmba. Ed. Saeculum . 1980.
2. Cronica ilustrat a omenirii. Editura Litera. Bucureti, 2011.
Prof. Liceul Tenologic. Costin Neniescu, Buzu


- ARTICOLE SI NOTE MATEMATICE -

Cnd raiunea altuia izbutete s m
conving pe deplin, devine propria mea raiune.
Jonathan Swift

2. Articole si note matematice



Demonstrarea unor inegaliti din Octogon Mathematical Magazine
(II)

de D.M. Btineu-Giurgiu, Neculai Stanciu i Titu Zvonaru

Acest articol este o continuare a celui din[1] i vom demonstra unele inegaliti enunate n [2].
1.
2 2 2
2 2 2
1 1 1
2
3 1 1 1
z y x
z y x
xyz
z y x z y x + +
+ +
>
+ +
+ + , 0 , , > z y x (vezi [2], Teorema 38, p. 101).
Demonstraie. Inegalitatea de demonstrat se scrie succesiv:

+ +
+ +
>
+ +
+ + + +
) (
) ( 2
) (
3 ) )( (
2 2 2
2 2 2 2 2 2
z y x xyz
x z z y y x
z y x xyz
xyz z y x zx yz xy

> + + + + + + + ) )( (
2 2 2 2 2 2 2 2 2
xz z x yz z y xy y x z y x
) )( ( 2
2 2 2 2 2 2
x z z y y x z y x + + + + > + + + +
_ _ _ _
3 2 2 3 4 4
y x y x xy y x

_ _ _ _ _ _ _
+ > + + + > +
2 3 4 4 2 2 3 2 2 3 2 2
2 2 2 2 y x xy y x z y x y x y x z y x
0 ) ( ) (
2 3 2
> + +
_ _
y x y x xy y x .
2.
4
4
4
4
4
4
3
3 4
|
|
.
|

\
|

+ +
> + + + t
z y x
xyzt t z y x , 0 , , > z y x (vezi [2], Teorema 6, p. 81).
Demonstraie. Pentru uurin notm
4 4 4 4
, , d t c z b y a x = = = = , i avem de demonstrat c
4
4 4 4 4
3
3 4 |
.
|

\
|

+ +
> + + + d
c b a
abcd d c b a . Putem presupune (eventual renotnd) c
{ } d c b a d , , , min = . Dac notm d a = o , d b = | , d c = , avem 0 , , > | o . Nu este foarte
greu de verificat identitatea:
+ + + + + + = + + + ] ) ( ) ( ) ( [ 2 4
2 2 2 2 2 2 2 4 4 4 4
o | | o | o d abcd d c b a
4 4 4 3 3 3
8 ) 3 ( 4 | o o| o| | o + + + + + + + d d , i atunci este suficient s demonstrm c
27
) (
4
4 4 4
| o
| o
+ +
> + + . O demonstraie pentru ultima inegalitate se poate obine cu
inegalitatea lui Radon, sau cu inegalitatea lui Cebev:
> + + + + > + + = + + ) )( (
3
1
3 3 3 3 3 3 4 4 4
| o | o | | o o | o
4 2 2 2 2
) (
27
1
) )( (
9
1
| o | o | o + + > + + + + > .
3. xyz
z y x
z y x y x z y x
>
+ +
+ + +
>
|
.
|

\
| + +
) ( 48
) 4 ( ) (
3
2 2
3
, 0 , , > z y x (vezi [2], Teorema 41, p. 102).


- ARTICOLE SI NOTE MATEMATICE -


Demonstraie. Inegalitatea din stnga se scrie succesiv
) 4 )( ( 3 ) ( 4 ) 4 ( ) ( 9 ) ( 16
2 2 2 4
z y x y x z y x z y x y x z y x + + + > + + + + + > + +
0 ) 2 ( 0 4 4 2 4
2 2 2 2
> + > + + + z y x yz xz xy z y x . Deoarece xy y x 4 ) (
2
> + ,
pentru inegalitatea din dreapta este suficient s demonstrm c
> + + + + + > + + 0 4 4 2 4 ) ( 12 ) 4 (
2 2 2 2
yz xz xy z y x z y x z z y x
0 ) 2 (
2
> + z y x .
4.
x
z
z
y
y
x
xyz
z y x
+ + s
+ +
3
, 0 , , > z y x (vezi [2], Teorema 39, p. 101).
Demonstraie. Notnd
3
a x = ,
3
b y = ,
3
c z = , avem de demonstrat c
5 2 2 2 5 2 2 2 5 6 3 6 3 3 6
c b a c b a c b a c b b a c a + + > + + , care rezult din adunarea urmtoarelor trei
inegaliti:
2 2 5
6 3 3 6 3 6
3
c b a
b a c a c a
>
+ +
;
2 5 2
6 3 6 3 6 3
3
c b a
c b b a b a
>
+ +
;
5 2 2
3 6 6 3 6 3
3
c b a
c a c b c b
>
+ +
.
5. > + + + + +
2 2 2 2 2 2
) ( ) ( ) ( ) ( ) ( ) ( t z t y z y t x z x y x

4
4 xyzt t z y x + + + > , 0 , , , > t z y x (vezi [2], Teorema 14, p. 90).
Demonstraie. Notnd
4
a x = ,
4
b y = ,
4
c z = ,
4
d t = , avem de demonstrat c
2 2 2 2 2 2 2 2 2 2 2 2 4 4 4 4
2 d c d b c b d a c a b a abcd d c b a + + + + + > + + + + .
Folosim inegalitatea din teorema 12 ([2]), adic
) )( ( 4 ) ( 3
3 3 3 3 4 4 4 4
d c b a d c b a abcd d c b a + + + + + + > + + + + , rescris sub forma
+ + + + + + + + > + + + +
3 3 3 3 3 3 3 3 4 4 4 4
4 2 2 2 2 bc c b ad d a ac c a ab b a abcd d c b a
3 3 3 3
cd d c bd d b + + + + . Rezult c este suficient s artm c
+ + + + + + + +
3 3 3 3 3 3 3 3
bc c b ad d a ac c a ab b a > + + +
3 3 3 3
cd d c bd d b
2 2 2 2 2 2 2 2 2 2 2 2
2 2 2 2 2 2 d c d b c b d a c a b a + + + + + > , care este echivalent cu
0 ) ( ) ( ) ( ) ( ) ( ) (
2 2 2 2 2 2
> + + + + + d c cd d b bd c b bc d a ad c a ac b a ab .
6. ) )( )( ( 27
) )( )( (
8
3
x z z y y x xyz
z y x
xyz x z z y y x
+ + + >
|
|
.
|

\
|
+ +
+ + + +
, 0 , , > z y x (vezi [2], Teorema 33,
p. 99).
Demonstraie. Deoarece ) )( ( ) )( )( ( zx yz xy z y x xyz x z z y y x + + + + = + + + + ,
inegalitatea de demonstrat se scrie succesiv:
+ + + + + + > + +
_ _ _
z y x z y x y x x z z y y x xyz zx yz xy
3 2 2 3 3 3 3
3 3 ( 8 ) )( )( ( 27 ) ( 8
+ + > + + > +
_ _ _ _ _
z y x z y x y x xyz xy y x xyz z y x
3 2 2 3 3 3 2 2 2 2 2
3 3 8 ) 2 ( 27 ) 6
0 ) 3 ( 5 ) 3 ( 3 6
2 2 2 3 3 3 2 2 3 2 2 2 3 3 2 2 2
> + + +
_ _ _ _
z y x y x z y x z y x z y x y x z y x ;
prima parantez este pozitiv conform inegalitii lui Schur (aplicat numerelor zx yz xy , , ), iar a
doua parantez este pozitiv conform inegalitii mediilor.

Bibliografie:

1. D.M. Btineu-Giurgiu, N. Stanciu, T. Zvonaru, Demonstrarea unor inegaliti din Octogon,
Sclipirea Minii, Nr. 12, 2013, 5-6.
2. Octogon Mathematical Magazine, Vol. 13, April, 2005.


- ARTICOLE SI NOTE MATEMATICE -

Aplicaii de analiza matematic
de prof. Florin Stnescu , Dmbovia
(II)

n cele ce urmeaz prezentm continuarea articolului din numrul 11 ce i propunea determinarea
unor funcii, definite, n special, prin relaii integrale, cu ajutorul inegalitilor lui Cebev i a lui Jensen.
Aplicaia 4. Determinai funciile :[0,1] f 1,care au urmtoarele proprieti:
a) f este derivabil de trei ori cu '''( ) 0, ( ) [0,1]; f x x > e
b) ' f este cresctoare si strict pozitiva;
c) ( ) ( )
( )
1
2
0
'(1) 2 (1) (0) '(1) 1.
'( )
dx
f f f f
f x
=
)
Mathproblems ,Volume 2, Issue 2 (2012)
Rezolvare: Din
( ) ( ) ( )
' '
1 1 1
2 2 2
0 0 0
'( ) ''( ) '( ) ''( ) 1 '( ) ''( )
,( ) [0,1] .(5)
'( ) '( ) '(1)
'( ) '( ) '( )
x f x xf x x f x xf x f x xf x
x dx dx dx
f x f x f
f x f x f x
| | | |
= e = =
| |
\ . \ .
) ) )

Considerm funcia

:[0,1] , ( ) '( ) ''( ) '( ) '''( ) 0 g g x f x xf x g x xf x = = s 1 g este descresctoare.
Cum funciile ( ) g x si
( )
2
1
'( ) f x
sunt descresctoare, conform ) ) a o ,avem
( ) ( )
( )
( )
1 1 1 1 1
2 2 2
0 0 0 0 0
1 '( ) ''( )
'( ) ''( ) (1) (0) '(1) '( )
'(1)
'( ) '( ) '( )
f x xf x dx dx
dx f x xf x dx f f f f x dx
f
f x f x f x
| |

= > = + =
|
\ .
) ) ) ) )


( )
( ) ( )
1
2
0
2 (1) (0) '(1)
'( )
dx
f f f
f x

)
(6). Relaia din enun mai poate scris sub forma
1
'(1) f
=
( )
( ) ( )
1
2
0
2 (1) (0) '(1)
'( )
dx
f f f
f x

)
, deci
1
'(1) f
=
( )
( ) ( )
1
2
0
1
2 (1) (0) '(1)
'(1)
'( )
dx
f f f
f
f x
>
)
.
Astfel,
( )
2
1
'( ) f x
sau g sunt funcii constante.
1.Dac '( ) , ( ) [0,1] ( ) , , , 0. f x a x f x ax b a b a = e = + e > 1 (7)
2.Dac
( )
0
( ) '( ) ''( ) , ( ) [0,1] '( ) ''( ) , ( ) [0,1] ( ) (0) '( )
x
g t c f t tf t c t f t tf t dt cx x f x f xf x = = e = e +
)
0
2 (0)
'( ) 2( ( ) (0)) '( ) '( ) ( ) , ( ) (0,1].
x
f
f t dt cx f x f xf x cx f x f x c x
x x
| |
= = = + e
|
\ .
)
Ultima
relaie reprezint o ecuaie diferenial liniar de ordinul nti: ' ( ) ( ) y p x y q x = + , cu soluia
( ) ( )
( ) ,
P x P x
y e q x e dx

=
)
unde ( ) P x reprezint o primitiva fixata a funciei . p Cum
2ln 2ln 2 2
2 2
(0) (0) 1 (0)
( ) 2ln ( )
2
x x
f f f c
P x x f x e c e dx x c dx x d
x x x x x

| | | | | |
= = = = + + =
| | |
\ . \ . \ .
) )

2
(0)
,( ) (0,1].
2
f
dx cx e x + + + e

Deoarece f este continu pe
2
(0)
[0,1] ( ) ,( ) [0,1].
2
f
f x dx cx e x = + + + e

(8).
Din (7) si (8) i respectnd condiiile din enun obinem c
2
( ) , 0, 0. f x ax bx c a b = + + > >
Aplicaia 5. Determinai funciile continue si cresctoare : 0, [0, ),
2
f
t (

(

cu urmtoarele proprieti:



- ARTICOLE SI NOTE MATEMATICE -

a)
2 2
cos (sin ) sin (cos ) 1, ( ) 0, ;
2
x f x x f x x
t (
+ > e
(

b)
2
0
( ) .
2
f x dx
t
t
s
)

Soluie. Avem c
( ) sin f x este o funcie cresctoare, iar
( ) cos f x descresctoare, deci ambele funcii sunt
integrabile. Aplicnd inegalitatea Cauchy-Schwarz, putem scrie:
( )
2
1 cos cos (sin ) sin sin (cos ) x x f x x x f x s + s
( ) ( ) ( )
( ) ( )
2 2
2 2 2 2 2 2
cos sin cos (sin ) sin (cos ) cos sin sin cos , x x x f x x f x x f x x f x
| |
+ + = +
|
\ .
( ) 0, .
2
x
t (
e
(

Integrnd pe 0,
2
t (
(

ultima inegalitate, putem scrie:
2 2
2 2
0 0
sin (cos ) cos (sin ) .
2
x f x dx x f x dx
t t
t
s +
) )
(9) .Pentru integrala
2
2
0
sin (cos ) x f x dx
t

)
cu schimbarea
de variabila
2
x t
t
= , obinem
2 2
2 2
0 0
sin (cos ) cos (sin ) . x f x dx x f x dx
t t
=
) )
Revenind in (9) avem
2
2
0
cos (sin )
4
x f x dx
t
t
s
)
.Folosind ca: sin , ( ) 0, ,
2
x x x
t (
s e
(

f cresctoare, deci
(sin ) ( ), ( ) 0, ,
2
f x f x x
t (
s e
(

si ) ) b o , avem:
2 2 2 2
2 2 2
0 0 0 0
2 2
cos (sin ) cos ( ) cos ( ) (
4 4 2 4
x f x dx x f x xdx f x dx
t t t t
t t t t
t t
s s s s =
) ) ) )
am mai folosit ca
2
cos x este descresctoare pe 0, )
2
t (
(

, rezulta ( ) , 0, ,
2
f x c x
t (
= e
(

iar din
2
0
( ) 0 1.
2
f x dx c
t
t
s s s
)

nlocuind f c = in relaia din enun
( )
2 2
sin cos 1 1 1 ( ) 1, ( ) 0, .
2
x x c c c f x x
t (
+ > > = = e
(


O alt aplicaie n care se folosete egalitatea lui Young i inegalitatea lui Jensen este urmtoarea problem
Aplicaia 6. Determinai funciile integrabile :[0, 2] f 1, pentru care exist a e1 astfel nct
( )
3
sin , ( ) [0,1] f x x ax x + = e si
2
0
5
( ) 2sin .
16
a
f x dx =
)

Bibliografie
1. Boboc N. Analiza matematica, Editura Universitii din Bucureti, 1999;
2. Ganga M. Teme si probleme de matematica, Editura Tehnica, Bucureti, 1991;
3. Mortici C. 600 de probleme de matematica pentru concursuri, Editura Gil, Zalau, 2001.
Prof. , coala erban Cioculescu, Geti, Dmbovia

tiina puin i face pe oameni pretenioi, n timp ce tiina mult i face modeti: aa dup
cum spicele goale i nal spre cer capetele lor trufae, n timp ce spicele pline, se apleac spre
pmnt sub greutatea lor.
Leonardo da Vinci



- ARTICOLE SI NOTE MATEMATICE -


A method for solving equations

by Mihly Bencze and Titu Zvonaru

Here, we present a method for solving algebraic equations, trigonometric, exponential, or combining
these types of equations.
We must to solve the equation: 0 ) ( = x F , I x e , R I c , (1),where 1 is the set of real numbers. We put
the equation to solve in the form: )) ( ( )) ( ( x h f x g f = , I x e , (2)
where : f J 1 is an injective function on J , and g respectively h are functions with values in J . By
the injectivity of the function f , the equation (2) becomes ) ( ) ( x h x g = , I x e , (3),which generally is easily
solved. The difficulty is in choosing the function f . Here we present some illustrate examples ([2]) as
follows:
Example 1. Solve in 1the equation:
1 cos 8 5 cos 2 cos 3 8
cos
cos 8 5 cos
log
3 cos 8 5 cos cos
3
3
3
+ + =
+
+
x x x
x
x x
x x x
.
Solution. Considering the injective function t t t f
t
+ + = 2 log ) (
3
, 0 > t , the equation to solve, it is
written successively
) cos 3 ( ) cos 8 5 (cos
3
x f x x f = + x x x cos 3 cos 8 5 cos
3
= +
0 cos 3 cos 2 cos 4 5 cos
2
= + x x x x 0 cos 3 ) 2 cos 1 ( cos 4 5 cos = + + x x x x
0 2 cos cos 4 cos 5 cos = + + x x x x 0 2 cos cos 4 2 cos 3 cos 2 = + x x x x
0 ) cos 2 3 (cos 2 cos = + x x x ( ) 0 cos 2 ) 3 cos 4 ( cos 2 cos
2
= + x x x x
0 ) 1 cos 4 ( 2 cos cos
2
= x x x ( ) 0 1 ) 2 cos 1 ( 2 2 cos cos = + x x x
0 ) 1 2 cos 2 ( 2 cos cos = + x x x .
Hence, 0 cos = x , or 0 2 cos = x , or
2
1
2 cos = x .
We obtain the solutions
)
`

e +
)
`

e +
)
`

e + e Z k k Z k k Z k k x t
t
t
t
t
t
6 4
2
2
.
Example 2. Solve in 1the equation:

2
log
) 2 (
) 4 3 2 (
5 9 3 3 2 3
2
3
2 2
2
2 3 5 3 5 4
2 4 2 4
+
+
+
+ +
+ = + + +
+ +
x
x
x
x x x
x x
x x x x

Solution. Considering the injective function
t t
t t f
2
3 3 3 2 ) ( + + = , 0 > t , the given equation is written
) 3 5 (
2
log
2 4
2
3
+ = |
.
|

\
|
+
x x f
x
x
f .
Therefore, it remains to solve the equation 3 5
2
log
2 4
2
3
+ =
+
x x
x
x
, 0 > x .
If ) 2 , 1 ( e x , then 1 3 5
2 4
< + x x and 1
2
log
3
1
2
2
3
2
>
+
>
+ x
x
x
x
;
If ) , 2 ( ) 1 , 0 ( e x , then 1 3 5
2 4
> + x x and 1
2
log
3
1
2
2
3
2
<
+
<
+ x
x
x
x
.
Yields that the only possible 1
1
= x and 2
2
= x which are indeed solutions.
This completes the proof.
Example 3. Solve in 1the equation:
2
3
log 10 3 9 2
2
2
2 4
+
= +
x
x
x x x .
Solution. We consider the injective function
2
2
2 2 log ) ( t t t t f + + = , 0 > t .



- ARTICOLE SI NOTE MATEMATICE -

Then the equation to solve is written as ) 3 ( ) 2 (
2
x f x f = + .
So, 0 2 3
2
= + x x , and we get the solutions 1
1
= x , 2
2
= x . The proof is complete.
Example 4. Solve in 1the equation:

6 4 3 2
2
3
6
3 42 2 135 22 121
1
11
log x x x x x
x
x x
+ =
+
+

Solution. Considering the injective function
2
6
3 2 log ) ( t t t t f + + = , 0 > t ,
the equation to solve is written as ) 6 6 ( ) 11 (
2 3
+ = + x f x x f .
So we must to solve the equation 6 6 11
2 3
+ = + x x x .
We get the solutions 1
1
= x , 2
2
= x , 3
3
= x . The proof is complete.
Example 5. Solve in 1the equation:
2 2 5
2 2
8 5
log
2
8 5
log 7
2
3
2
2
2
+ =

+
+

+
+ + x x
x
x x
x
x x
x .
Solution. Considering the injective function t t t t f + + =
3 2
log log ) ( , the equation to solve
becomes ) 2 2 ( ) 8 5 (
2
= + x f x x f .
So we must to solve the equation 2 2 8 5
2
= + x x x ,
which by squaring becomes 0 72 84 41 10
2 3 4
= + + x x x x
0 ) 8 4 ( ) 3 (
2 2
= + x x x . Hence 3 = x , and we are done.
Example 6. Solve in 1the equation: x
x
x
x
x x 3 1
3
1
log
3
1
log 3
2
3
2
2
3
= + |
.
|

\
|
+ + |
.
|

\
|
+ +
Solution. Considering the injective function t t t t f + + =
3 2
log log ) ( , the given equation
becomes ) 3 ( ) 1 3 (
3
x f x f = + . So it remains to solve the equation x x 3 1 3
3
= + , which has all roots real for
e.g.
|
.
|

\
|
e
2
1
, 0
1
x ,
|
.
|

\
|
e 1 ,
2
1
2
x . So, you find the solutions using Cardano s formulas.
Example 7. Solve in 1the equation:

6 4 3 2
3
28 216 226 6 819
2
3
6
log x x x x x
x
x
+ + =
|
.
|

\
|
+ .
Solution. If we consider the injective function t t t t t f + + + =
2 3
3
log ) ( ,
the equation it is written in the form ) 6 ( ) 9 (
2
x f x f = + . So, x x 6 9
2
= + , i.e. 3 = x , and we are done.
Example 8. Solve in 1the equation: 0 2 cos ) cos sin 3 ( 2 2
2 2 sin cos
2 2
= + x x x
x x

Solution. We have ( ) x x x x x x x x 2 cos cos sin ) cos (sin 3 2 cos ) cos sin 3 (
2 2 2 2 2 2 2
+ =
) sin )(cos cos sin 5 cos 3 sin 3 (
2 2 2 2 4 4
x x x x x x + + =
x x x x x x
4 2 2 4 6 6
cos sin 2 cos sin 2 sin 3 cos 3 + =
) sin 1 ( sin 2 sin 3 ) sin 1 ( cos 2 cos 3
2 4 6 2 4 6
x x x x x x + =
x x x x
4 6 4 6
sin 2 sin cos 2 cos + = .
If we consider the injective function
2 3
2 2 ) ( t t t f
t
+ + = , then the given equation becomes
) (sin ) (cos x f x f = .Therefore, x x sin cos = , which yields t
t
k x + =
4
, Z k e .
REFERENCES
[1] M. Bencze, Utilizarea injectivitii funciilor n rezolvarea ecuaiilor, Recreaii Matematice, 1(2013), 27-
28.
[2] Octogon Mathematical Magazine, 1993-2013.



- ARTICOLE SI NOTE MATEMATICE -



Inegaliti ciclice obinute cu funcii convexe

de prof. Marius Drgan, Horaiu Stoian, Bucureti

Scopul acestui articol este de a demonstra c inegalitatea ciclic:

_ [ _
+
+
+
+
>
1
1
1
1 2 1
n n
x x x x
| o | o
| o
(1)
este adevrat pentru orice
n
x x x ,..., ,
2 1
pozitive, 2 > n , ) , 0 ( , e | o astfel nct | o n > .
Substituind
| o +

i
i
x
x , n i , 1 = , i considernd funcia convex
x
e x f = ) ( , x e1, inegalitatea (1)
este echivalent sub forma mai general cu:
Teorem. Fie ) , 0 ( ,..., ,
2 1
e
n
x x x , 2 > n , ) , 0 ( , e | o astfel nct | o n > , i : f 1 1 o
funcie convex Atunci are loc inegalitatea:
>
|
|
.
|

\
|
+
+
+ +
|
|
.
|

\
|
+
+
+
|
|
.
|

\
|
+
+
| o
| o
| o
| o
| o
| o
1 3 2 2 1
...
x x
f
x x
f
x x
f
n

|
.
|

\
|
+
+ + +
+ + |
.
|

\
|
+
+ + +
+ |
.
|

\
|
+
+ + +
>
1
2 ...
...
1
... 2
1
... 2
2 1 2 1 2 1
n
x x x
f
n
x x x
f
n
x x x
f
n n n
(2).
Demonstraie: Din inegalitatea Jensen cutam ) 1 , 0 ( ,..., ,
2 1
e
n
o o o astfel nct:
>
|
|
.
|

\
|
+
+
+ +
|
|
.
|

\
|
+
+
+
|
|
.
|

\
|
+
+
| o
| o
o
| o
| o
o
| o
| o
o
1 3 2
2
2 1
1
...
x x
f
x x
f
x x
f
n
n

|
.
|

\
|
+
+ + +
=
|
|
.
|

\
|
+
+
+ +
+
+
+
+
+
>
1
... 2 ) (
...
) ( ) (
2 1 1 3 2 2 2 1 1
n
x x x
f
x x x x x x
f
n n n
| o
| o o
| o
| o o
| o
| o o

i analoagele
>
|
|
.
|

\
|
+
+
+ +
|
|
.
|

\
|
+
+
+
|
|
.
|

\
|
+
+

| o
| o
o
| o
| o
o
| o
| o
o
1
1
3 2
1
2 1
...
x x
f
x x
f
x x
f
n
n n

|
.
|

\
|
+
+ + +
=
|
|
.
|

\
|
+
+
+ +
+
+
+
+
+
>

1
... 2 ) (
...
) ( ) (
2 1 1 1 3 2 1 2 1
n
x x x
f
x x x x x x
f
n n n n
| o
| o o
| o
| o o
| o
| o o

>
|
|
.
|

\
|
+
+
+ +
|
|
.
|

\
|
+
+
+
|
|
.
|

\
|
+
+
| o
| o
o
| o
| o
o
| o
| o
o
1
1
3 2
3
2 1
2
...
x x
f
x x
f
x x
f
n

|
.
|

\
|
+
+ + +
=
|
|
.
|

\
|
+
+
+ +
+
+
+
+
+
>
1
2 ... ) (
...
) ( ) (
2 1 1 1 3 2 3 2 1 2
n
x x x
f
x x x x x x
f
n n
| o
| o o
| o
| o o
| o
| o o
(3)
Adunnd inegalitile (3) vom obtine inegalitatea (2). Ne rmne s demonstrm c
n
o o o ,..., ,
2 1

sunt bine determinate. Prin identificarea coeficienilor din (3) obinem sistemul

1
) ( 2
1
+
+
= +
n
n
| o
|o oo ;
1
2 1
+
+
= +
n
| o
oo |o ;
1
3 2
+
+
= +
n
| o
oo |o ;...;

1
1
+
+
= +

n
n n
| o
oo |o (4)
Deoarece determinantul sistemului 0 ) ( = =
n n
d | o , rezult c sistemul este compatibil
determinat. Adunnd egalitile (4) rezult 1 ...
2 1
= + + +
n
o o o .


- ARTICOLE SI NOTE MATEMATICE -

Din:
1
1
+
+
= +

n
k k
| o
oo |o , n k , 2 = rezult |
.
|

\
|
+
=
+

1
1
1
1
1
n n
a
k k
o
o
|
, sau
|
.
|

\
|
+
|
.
|

\
|
=
+

1
1
1
1
1
1
n n
a
i
i
o
o
|
, n i , 1 = . Pentru n i = obinem
|
.
|

\
|
+

|
.
|

\
|
=
+

1
1
1
1
1
1
n n
a
n
n
o
o
|
, iar apoi din prima ecuaie a sistemului rezult
1
1
1
1
1
1
1
) 1 (
) 1 (
1
1
) 1 (
) ( 2

|
.
|

\
|
+

+ =
+

+
+
n
n
n
n
n
n n n o
|
o
o
|
o
|
o
|
| o
, de unde rezult
] ) ( )[ 1 (
) ( 2
1
1
n n
n n n
n
a
| o
| | o o
+
+
=

. Artm c 1 0
1
< < a .
Cazul 1. = n par.
) )( 1 (
) ( 2
] ) ( )[ 1 (
2
1 1 1
1
n n
n n n
n n
n n n
n n
a
| o
| | o o
| o
| | o o
+
+
=
+
+
=

. Evident | | o > > n .
Rezult 0
1
> o . Artm c 1
1
< o echivalent cu
n n n
n n o | | o ) 1 (
1
< +

. Dar,
n
o
| s , de unde
n
n n
n n
n
n
n
n
o o
| | o + < +
1
, deci este suficient s artm c
n
n
n n
n
n
n
n
o
o o
) 1 ( < +
1
1 1
1
< +

n
n n
n
. Dar, 1 2
1 1
1
s < +

n
n n
n
, deci pentru = n par 1 0
1
< < a .
Cazul 2. = n impar. 0
) )( 1 (
2
1
1
>
+ +
+ +
=

n n
n n n
n
a
| o
| | o o
. Artm c 1
1
< o echivalent cu
Artm c echivalent cu
n n n
n n o | | o ) 1 (
1
+ <

. Dar,
n n
n
n n
o o
o | o = s
1 1

Artm c
n n
n
n n
n
o |
o
) 1 ( + < , inegalitate adevrat deoarece 1
1
< n
n
.
n continuare demonstrm inductiv c 1 0 < <
i
a , n i , 1 = .
Presupunem 1 0
1
< <
i
a , adevrat. Artm 1 0 < <
i
a . Deoarece | | < <
1
0
i
a , deducem c
| oo
| o
<
+
+
<
i
n 1
0 . Rezult 1
1
) 1 ( ) 1 (
< =
+
+
s
+
+
<
n n
n
n
i
o
o
o
o
| o
o .
De asemenea 0
) 1 ( 1
1
>
+

=
|
.
|

\
|

+
+
>
n
n
n
i
o
| o
|
| o
o
o . Deci, 1 0 < <
i
a , n i , 1 = .
Corolar. Fie ) , 0 ( ,..., ,
2 1
e
n
x x x , 2 > n , ) , 0 ( , e | o astfel nct | o n > . Atunci are loc
inegalitatea: ( ) 1
2 1
1 1
2
1
1 1 3 2 2 1
... ... ... +
+
+
+
+
+
+
+
|
|
.
|

\
|
+ + + > + + + n
n
n
n
n n
n
x x x x x x x x x x x x
| o
| o | o | o
| o | o | o
.
Aplicaii:
1)pentru 3 , 1 , 3 = = = n | o , obinem inegalitatea
2
3 2 1 3
2
2 1 3 2
2
1 1
3
3 3
3
2 2
3
1
x x x x x x x x x x x x x x x + + > + + ;
2)pentru 4 , 1 , 4 = = = n | o , obinem inegalitate ) (
4 3 2 1 4 3 2 1 1
4
4 4
4
3 3
4
2 2
4
1
x x x x x x x x x x x x x x x x + + + > + + +





- ARTICOLE I NOTE MATEMATICE -


Probleme de calendar
de prof. Ionel Tudor, Giurgiu

I. Matematica i zilele sptmnii

n anii 1886-1887 , reverendul Cristian Zeller a stabilit o formul matematic pentru aflarea zilei
sptmnii unei date oarecare.
Pentru a nelege formula lui Zeller , amintim c partea ntreag a unui real x , se noteaz [x] i
reprezint cel mai mare numr ntreg care nu depete pe x . Mai exact , dac scrierea zecimal a lui x este x
=
0
a ,
3 2 1
a a a unde
0
a este numr natural iar , , ,
3 2 1
a a a sunt cifre , atunci [x]=
0
a .
Dac z reprezint ziua din luna l a unui an N , iar n este numrul format din ultimile dou cifre ale lui
N i m este numrul format din celelalte cifre ale lui N , atunci
S =
( )
=
(

+
+
(

+
(

+ +
10
1 26
4 4
2
l n m
m n z M7+r , unde r < 7 indic ziua sptmnii,
corespunztor lui 0-smbt , 1-duminic , 2-luni , 3-mari , 4-miercuri , 5-joi , 6-vineri .
Lunile ianuarie i februarie se consider ca a 13-a i a 14-a lun a anului precedent.
Aceast formul este valabil n calendarul gregorian , adic cel folosit n mod obinuit, pe stil nou
Pentru calendarul iulian , adic cel pe stil vechi , formula este:
S =
( )
=
(

+
+
(

+ +
10
1 26
4
2
l n
m n z M7+r , unde r < 7.
Exemple:
1. n ce zi a sptmnii a czut data de 2 iulie 1504 ?
Este data (,,pe stil vechi) la care s-a stins din via domnitorul Moldovei , tefan cel Mare.
z=2 , l=7 , N=1504 , m=15 , n=04=4. Deci,
S=2+4-15-2 +
(


+
(

10
8 26
4
4
= - 10+20 = 10 = 3 7 1 + , deci r = 3 care corespunde zilei de mari.
Pe acopermntul de mormnt , de la Mnstirea Putna , al marelui voievod , lucrat din porunca fiului su
Bogdan al III-lea Vlad , este scris :
,,Io Bogdan Voievod din mila lui Dumnezeu , domnul rii Moldovei , a nfrumuseat i a acoperit cu acest
acopermnt mormntul tatlui su , Io tefan Voievod , cel care a domnit n ara Moldovei 47 de ani i trei
luni , care s-a strmutat la lcaul de veci n anul 7012=1504 luna iulie , ziua 2 de mari ,n ceasul al
patrulea din zi
2. n ce zi a sptmnii a czut data de 13 august 1595 (pe stil vechi , iar pe stil nou 23 august 1595) ?
Este data victoriei obinute de marele voievod Mihai Viteazul , n lupta de la Clugreni , mpotriva armatei
turceti condus de Sinan Paa.
z=13 , l=8 , N=1595 , m=15 , n=95 iar S=13+95-15-2+ =
(


+
(

10
9 26
4
95
108-17+23+23=137=7 19+4,deci
r = 4 care corespunde zilei de miercuri.
Cu formula pentru calendarul gregorian se obine tot S=137 i aceeai zi de miercuri.
Marele nostru patriot i revoluionar de la 1848, Nicolae Blcescu n opera sa ,,Romnii supt Mihai -
Voievod Viteazul consemna :
,,n sfrit soarele veni s lumineze aceast mare zi de miercuri 13/23 august , menit a fi briliantul cel mai
strlucit al cununei gloriei romne.
3. S vedem n care zi a sptmnii s-a petrecut Unirea Principatelor Romne la 24 ianuarie 1859.
z=24, l=13, N=1859 , n=58 , m=18
18 58 26 14
24 58 36 46 4 14 36 100 7 14 2
4 4 10
S
( ( (
= + + + + = + + + = = +
( ( (

,
deci r=2 iar actul unirii s-a petrecut ntr-o zi de luni.
4. n ce zi a sptmnii a fost data de 1 Decembrie 1918 ? (Ziua ,, Marii Uniri)
z=1, l =12 , N=1918 , m=19 , n =18 i obinem :




- ARTICOLE I NOTE MATEMATICE -

S=1+18-38+ 1 7 3 22 33 4 4 19
10
13 26
4
18
4
19
+ = = + + + =
(


+
(

+
(

, deci r = 1 i corespunde zilei de


duminic.
5. Menionm c formulele rmn valabile i pentru anii biseci.
S vedem n ce zi din sptmn a czut 29 februarie, n anul bisect 2012.
z=29,l=14,N=2012,m=20,n=11
20 11 26 15
29 11 40 5 2 39 46 7 6 4
4 4 10
S
( ( (
= + + + + = + + = = +
( ( (

,
deci r = 4 care corespunde zilei de miercuri.

II. Matematica i Patele

Problema aflrii datei unei anumite zile este legat de unele fenomene astronomice. De exemplu,
data Duminicii Patelui , n religia ortodox pe stil vechi, a preocupat i pe marele matematician i
astronom german Karl Gauss.
n anul 325 , la Conciliul de la Niceea , la care au participat i Sfinii mprai Constantin i Elena, s-a
stabilit ca prima zi de Pati s fie prima duminic dup luna plin, dup ziua echinociului de primvar
(21 martie), sau ntr-o zi urmtoare acestuia. Aceast zi cade la
date diferite n diferii ani , repetndu-se ntr-un ciclu de 28 ani.
Cum fazele lunii se repet cu o perioad de 19 ani , Duminica Patelui ortodox pe stil vechi, i schimb data
n calendar la un interval de 28 19 532 = ani. Stabilirea ei
revine la rezolvarea n numere ntregi a unor ecuaii liniare diofantice , sau mai pe scurt , dup procedeul lui
Gauss, la folosirea resturilor unor mpriri. Cum anume?
Iat rspunsul :
Pentru a afla data x a primei zi de Pati ortodox pe stil vechi ntr-un anumit an n , trebuie s calculm
resturile unor mpriri i anume :
1
mod19 r n = (adic restul mpririi lui n la 19 ) ;
n r =
2
mod 4 (restul mpririi lui n la 4) ;
n r =
3
mod 7 (restul mpririi lui n la 7);
( ) 15 19
1 4
+ = r r mod 30 (restul mpririi lui 19
1
r +15 la 30) i
( ) 6 6 4 2
4 3 2 5
+ + + = r r r r mod 7 (restul mpririi lui 6 6 4 2
4 3 2
+ + + r r r la 7 ).
Calculm apoi . 4
5 4
+ + = r r x
Dac 30 s x , atunci aceast dat cade ntr-o duminic din luna aprilie .
Dac x>30 , atunci cifra unitilor lui x reprezint data Duminicii Patelui n luna mai.
Patele ortodox pe stil vechi cade cel mai devreme pe 4 aprilie i cel mai trziu pe 8 mai.
Patele catolic pe stil nou cade cel mai devreme pe 22 martie i cel mai trziu pe 25 aprilie.
Exemple:
1.n ce dat de duminic va fi Patele in 2014?
0 19 mod 2014
1
= = r ; 2 4 mod 2014
2
= = r ; 5 7 mod 2014
3
= = r ;
( ) ( ) 15 30 mod 15 0 19 30 mod 15 19
1 4
= + = + = r r ;
( ) ( ) 1 7 mod 120 7 mod 6 15 6 5 4 2 2 7 mod 6 6 4 2
4 3 2 5
= = + + + = + + + = r r r r
20 4 1 15 4
5 4
= + + = + + = r r x <30 , deci , n 2014 , Patele ortodox cade duminic 20 aprilie(la fel ca i
Patele catolic pe stil nou) .
2.S gsim n ce dat va fi Patele ortodox din anul 2078.
7 19 mod 2078
1
= = r ; 2 4 mod 2078
2
= = r ; 6 7 mod 2078
3
= = r ;
( ) ( ) ( ) 28 30 mod 148 30 mod 15 133 30 mod 15 7 19 30 mod 15 19
1 4
= = + = + = + = r r ;
( ) ( ) 6 7 mod 202 7 mod 6 28 6 6 4 2 2 7 mod 6 6 4 2
4 3 2 5
= = + + + = + + + = r r r r ;
38 4 6 28 4
5 4
= + + = + + = r r x >30 i n 2078 , Patele ortodox cade n duminica de 8 mai.




- ARTICOLE I NOTE MATEMATICE -


Interesant este faptul c 2078 , este singurul an n perioada 2000-2100 , cnd Patele ortodox
cade cel mai trziu , anume n 8 mai.

Prezentm n continuare data Sfintelor Pati ( ortodox stil vechi) pe perioada anilor 2008-2025 (Pascalia)
: 2008-27 aprilie; 2014-20 aprilie ; 2020-19 aprilie ;
2009-19 aprilie; 2015-12 aprilie ; 2021- 2 mai ;
2010- 4 aprilie; 2016- 1 mai ; 2022-24 aprilie ;
2011-24 aprilie; 2017-16 aprilie ; 2023-16 aprilie ;
2012-15 aprilie; 2018- 8 aprilie ; 2024- 5 mai ;
2013- 5 mai ; 2019-28 aprilie ; 2025-20 aprilie.

Bibliografie: 1. Colecia Revistei de Matematic din Timioara(R.M.T.)
2. Wikipedia.
Prof., Lic. Tehn. ,,Mihai Viteazul Clugreni, Giurgiu



O metod de calcul al unor integrale definite

de Constantin Rusu, Rmnicu Srat

Scopul acestui articol este de a da o metod de calcul al unor integrale definite. Acest metod a fost
obinut prin generalizarea unor probleme publicate n Gazeta Matematic. n final se propun probleme care
se rezolv aplicnd metoda dat.
Teorem. Dac | | R b a f , : , | |
*
, : R b a g , | | R b a h , : sunt funcii continue cu
) ( ) ( ) ( x h x s f x f = + , ) ( ) ( x s g x g = , | | b a x , e , b a s + = , atunci:
dx
x g
x h
dx
x g
x f
b
a
b
a
) )
=
) (
) (
2
1
) (
) (
.
Demonstraie. Cu subsituia t s x = , avem succesiv: =

= =
) )
) (
) (
) (
) (
) (
dt
t s g
t s f
dx
x g
x f
I
a
b
b
a

) ) ) ) )
= = =

=
b
a
b
a
b
a
b
a
b
a
dx
x g
x h
I dx
x g
x h
I dt
t g
t f
dt
t g
t h
dt
t g
t f t h
) (
) (
2
1
) (
) (
2
) (
) (
) (
) (
) (
) ( ) (
.
Observaie. Metoda este eficient dac se poate calcula
)
b
a
dx
x g
x h
) (
) (
.
Aplicaii:
A1. Cazul 1 ) ( = x g . Calculai dx tgx I
)
+ =
4
0
) 1 ln(
t
(N. Boboc i I. Colojoar, Manual de Analiz
Matematic, 1980)
Soluie. Considerm, ) 1 ln( ) ( tgx x f + = , R f
(

4
, 0 :
t
,
4
t
= s i avem:
2 ln
4
1 ln ) 1 ln(
4
) ( =
(

|
.
|

\
|
+ + + =
|
.
|

\
|
+ x tg tgx x f x f
t t
. Aplicnd teorema, obinem:
8
2 ln t
= I .
A2. Cazul k x h = ) ( . Calculai dx
x
x
x
x x
I
)

+
+
=
1
0
2
cos
sin
2 1
1
t
t
.




- ARTICOLE I NOTE MATEMATICE -
Soluie. Lum x
x
x x
x f t sin
2 1
1
) (
+
+
= , | | R f 1 , 0 : , x x g t
2
cos ) ( = , | |
*
1 , 0 : R g i
avem x x f x f x h t sin 2 ) 1 ( ) ( ) ( = + = . Din teorem, obinem
t
2
= I .
A3. Cazul k x h = ) ( .
3.1. Calculai dx
x
x
I
)

+
=
1
1
2
1
arccos
(Problema 22430, din G.M-B/1991, autor Alexandru
Constantinescu, Bucureti).
Soluie. Deoarece t = + ) arccos( arccos x x , cu teorema obinem:
( )
4
1 1
2 2 1
1
2 1
arccos
2
1
1
1
1
2
1
1
2
t t t t
= + = =
+
=
+
=


) )
arctg arctg arctgx dx
x
dx
x
x
I .
3.2. Fie funcia R R f : , continu pe 1 cu proprietatea c k x f x f = + ) ( ) ( , R x e .
S se calculeze
)

=
4
4
2
cos
) (
t
t
dx
x
x f
I (Problema 24132 din G.M.-B, 5-6/1999, autor Vasile Gorgot, Rm.
Vlcea).
Soluie. Aplicnd teorema avem: k dx
x
k
dx
x
x f
I = = =
) )

4
4
2
4
4
2
cos
1
2 cos
) (
t
t
t
t
.
3.3. Calculai:
)
+ +
=
2
0
cos sin 1
t
dx
x x
x
I . (Publicat n G.M.-B, 12/2002, autor Manuela Prjea,
Drobeta Tr. Severin).
Soluie. Cu teorema avem
) )
+ +
=
+ +
=
2
0
2
0
cos sin 1
1
4 cos sin 1
t t
t
dx
x x
dx
x x
x
I i de aici calculul
integralei se face cu substituia t
x
tg =
2
.
Obinem 2 ln
4
) 1 ln(
4 1 4
1
0
t t t
= + =
+
=
)
t
t
dt
I .
3.4. S se calculeze:
)
+
=
1
0
2
1
arcsin
dx
x x
x
I (Problema 25332 din G.M.-B, nr. 6/2005, autor Nicolae
Pavelescu, Rm. Vlcea).
Soluie. Deoarece
2
1 arcsin arcsin
t
= + x x rezult
2
t
= k i aplicnd teorema gsim:
=
|
|
.
|

\
|
+ |
.
|

\
|

=
+
=
+
=
) ) )
dx
x
x x
dx
x x
x
I
1
0
2
2
1
0
2
1
0
2
2
3
2
1
1
4 1
1
4 1
arcsin t t

3 6 3
1
3
1
3 2 3
1 2
3 2
2
1
0
t t t
=
|
|
.
|

\
|
+ =

= arctg arctg
x
arctg .



- ARTICOLE I NOTE MATEMATICE -


Generalizarea unei probleme date la
Olimpiada Balcanic de Matematic
pentru juniori , Antalya-Turcia 21-26 iunie 2013

Prof. erban George-Florin, Brila

La aceast olimpiad de matematic a fost propus urmtoarea problem :

Determinai toate perechile ordonate de numere naturale nenule (a,b) pentru care numerele
3
a b 1
a 1

+
i
3
b a 1
b 1
+

sunt simultan numere naturale nenule .


Propun generalizarea acestei probleme :
Determinai toate perechile ordonate de numere naturale nenule (a,b) pentru care numerele
n
a b 1
a 1

+
i
n
b a 1
b 1
+

sunt simultan numere naturale nenule , unde n este numr natural impar.
Soluie : Folosesc formulele :
1 2
1
... 1
1
n
n n
a
a a a
a

+
= + + e
+
N , n impar i aeN .
1 ....
1
1
2 1
+ + + + =


a a a
a
a
n n
n
,
1
1
1
) 1 (
1
1 ) 1 (
1
1
+
+

+
+
=
+
+
=
+

a
b
a
b a
a
b b a
a
b a
n n n

Dar
( 1)
1
n
a b
a
+
e
+
N , atunci
1
1
b
a
+
e
+
N
1
1
1
) 1 (
1
1 ) 1 (
1
1

+
+

+ +
=

+
b
a
b
a b
b
a a b
b
ab
n n n
, dar
( 1)
1
n
b a
b

N atunci
1
1
a
b
+
e

N i
1
1
b
a
+
e
+
N
rezult c i produsul lor este numr natural adic
1
1
b
b
+
e

N ,
1 1 2 2
1
1 1 1 1
b b
b b b b
+
= + = + e

N ,
2
1 b
e

N , { } { } 1 1, 2 2, 3 b b e e .
Dac b=2 atunci
2 1
1
n
a
a

e
+
N i 2 1
n
a + eN (A)
2 1 2( 1) 3 2( 1) 3
1 1 1 1
n n n
a a a
a a a a
+ +
= = e
+ + + +
N , dar
2( 1)
1
n
a
a
+
e
+
N
rezult c { } { } 1 1, 3 0, 2 a a + e e , a nenul , deci (2,2) este soluie.
Dac b=3 atunci N
a
a
n
e
+

1
1 3
i N
a
n
e
+
2
1 3
, N
a a
a
a
a
a
a
n n n
e
+

+
+
=
+
+
=
+

1
4
1
) 1 ( 3
1
4 ) 1 ( 3
1
1 3
,
dar
3( 1)
1
n
a
a
+
e
+
N rezult c
4
1 a
e
+
N , { } { } 1 1, 2, 4 0,1, 3 a a + e e , a nenul deci (1,3) ,(3,3) sunt
soluii. Dac a=3 , verificm i cealalt condiie
3 3 1
2
n
+
eN
1
3 1 (2 1) 1
1
2 2
n
u
u
+
+ + +
= = + eN. Dac
a=1 si b=3 , verificm i cealalt condiie
n
3 1 2v 1 1
v 1
2 2
+ + +
= = + eN , (A) . Analog se analizeaz i
cazul n=1 .
S={ (1,3) ,(3,3) , (2,2) } .

Bibliografie : Olimpiada Balcanic de Matematic pentru juniori ,Antalya-Turcia 2013

Prof., Liceul Pedagogic D. P . Perpessicius Brila

- ARTICOLE I NOTE MATEMATICE -

Comentarii despre cteva probleme din Gazeta Matematic
de Nela Ciceu, Bacu i Roxana Mihaela Stanciu, Buzu

n aceast not dorim s prezentm cteva comentarii i soluii inedite pentru unele probleme aprute
n G.M.-B nr. 5/2013, ale cror soluii au fost publicate n nr. 11/2013.
E:14495. Determinai numerele naturale x i numerele ntregi y , prime ntre ele, tiind c
xy x
y

2
2
5

este numr ntreg. ( D.M. Btineu-Giurgiu i Neculai Stanciu)

Deoarece 1 ) , ( = y x , este evident c perechea ) 0 , 1 ( nu este soluie. Iat o soluie ceva mai direct:
Cum 1 ) , ( = y x , rezult 1 ) , ( = y y x i atunci
) (
5
2
y x x
y

este numr ntreg dac i numai


dac
) (
5
y x x
este numr ntreg. Obinem posibilitile 1 ) ( = y x x ,
1 ) ( = y x x , 5 ) ( = y x x , 5 ) ( = y x x . Din prima relaie deducem
0 , 1 = = y x care nu convine, iar din celelalte trei relaii rezult soluiile ) 2 , 1 ( , ) 4 , 1 ( , ) 4 , 5 ( , ) 6 , 1 ( , ) 6 , 5 ( .
E:14496. Artai c fracia
37 792
481 941
73 297
184 149
+
+
= F este reductibil.
Grigore Dumitru
Soluia din revist este scurt i uor de urmrit:
Putem scrie
37
37
37 37
37 37
37 792
481 941
1 1
1 1
) 1 37 2 ( ) 1 37 8 (
) 1 37 5 ( ) 1 37 4 (
M
M
M M
M M
F =
+ +
+ +
=
+ +
+ +
= ,
deci fracia se poate simplifica cu 37.
Un elev care nu a rezolvat problema i vede aceast soluie se ntreab n mod natural: am neles,
dar cum trebuia s m gndesc la numrul 37?
Folosind o descompunere n factori, tim c dac a i b sunt numere naturale, iar t e un numr
natural impar, atunci
t t
b a + se divide cu b a + . Deoarece avem puteri diferite, cel mai simplu mod de a
folosi acest rezultat este s alegem 1 = b .
Scriind 1 184 1 149 1 184 1 149 184 149
481 941 481 941 481 941
+ + = + + = + , rezult c avem
posibilitile:
- 1 149
941
se divide cu 148 i 1 184
481
+ se divide cu 185;
- 1 149
941
+ se divide cu 150 i 1 184
481
se divide cu 183.
Cum
2
5 3 2 150 = , 61 3 183 = , 37 2 148
2
= , 37 5 185 = , putem ncerca s vedem dac fracia dat
se simplific cu 3 sau cu 37. Deoarece
792
297 se divide cu 3, dar
37
73 nu se divide cu 3, ne rmne doar
varianta 37. Avem

74 296
185 148
37 792
481 941
1 73 1 297
1 184 1 149
M M
M M
F
+
+
=
+ +
+ +
= i cum 37 8 296 = , 37 2 74 = , rezult c fracia se
simplific cu 37.
E:14497. Fie triunghiul ABC ( AC AB < ),
0
90 ) ( = ZA m i AE ( bisectoarea unghiului A
( ) (BC E e ). Punctul D se afl pe ) ( AC astfel nct dreptele DE i BC sunt perpendiculare. Dac
{ } AB DE M = i { } MC BD N = , demonstrai c triunghiul BNC este isoscel.
Cosmin Manea i Drago Petric
Prezentm dou soluii, diferite de cea publicat.
I. Triunghiurile CDE i ABC sunt asemenea (sunt dreptunghice i au un unghi ascuit comun) i atunci


- ARTICOLE I NOTE MATEMATICE -
A
B
C
D
M
N
E
A
B
C
D
c
a
b
A
'
D'


AC
AB
EC
DE
AC
EC
AB
DE
= = , (1).
Analog, din asemnarea triunghiurilor MBE i ABC
obinem

AC
AB
ME
BE
AB
BE
AC
ME
= = , (2).
Folosind teorema bisectoarei avem

AC
AB
EC
BE
= , (3).
Din (1), (2), (3) rezult c ED BE = i ME CE = i
atunci din triunghiurile dreptunghice BDE i CME
deducem c
0
45 ) ( ) ( = Z = Z ECM m EBD m , adic
triunghiul BCN este dreptunghic isoscel.
II. Din BM AC i BC ME rezult c D este ortocentrul triunghiului BCM , deci
0
90 ) ( = ZBNC m .
Folosind patrulaterul inscriptibil BEDA, deducem c
0
45 ) ( ) ( = Z = Z EAD m DBE m ; rezult c
triunghiul BNC este dreptunghic i are un unghi de
0
45 , deci este isoscel.
E:14500. S se arate c un patrulater cu lungimile laturilor 6 , 5 , 4 respectiv x nu poate avea
aria egal cu 2013 , oricare ar fi x numr real pozitiv.
Gabriel Sitaru
Prezentm o soluie care nu folosete faptul c patrulaterul de arie maxim cu laturi de lungimi date
este inscriptibil, i nici formula pentru aria unui patrulater inscriptibil.

Notm cu ] ... [ Z XY aria poligonului Z XY... .
Considerm patrulaterul ABCD, cu laturile
c CD b BC a AB = = = , , . Fie A' proiecia punctului
A pe dreapta BC i D' proiecia punctului D pe
dreapta AC .
Din inegalitatea triunghiului avem
BC AB AC + < .
Obinem succesiv:
[ ] [ ] [ ] ABCD ABC ACD = + =

2 2
BC AA AC DD ' '
+ s

2 2
BC AB AC DC
s + (


( )
2 2 2
BC AB AB BC DC ab bc ca + + +
< + = .
Deoarece am obinut o expresie simetric n c b a , , , rezult c oricare ar fi ordinea laturilor,
avem
2
] [
ca bc ab
ABCD
+ +
< .
Pentru { } { } 6 , 5 , 4 , , = c b a obinem 37 ] [ < ABCD ; cum 2013 37 < , rezult c un patrulater cu
laturile x , 6 , 5 , 4 nu poate avea aria egal cu 2013 .




- PROBLEME REZOLVATE -


Nu am euat. Doar am gsit 10000 de situaii care nu merg..
Thomas Edison


3. Probleme rezolvate


nvmnt primar

P:282. Determinai numerele naturale de dou cifre tiind c diferena dintre suma cifrelor i 7 este
egal cu produsul cifrelor. Prof. Gheorghe Drstaru, Buzu
Rezolvare: Fie numrul cutat de forma ab . Din problem rezult
7 (1 ) (1 ) 6 (1 )( 1) 6 a b a b a b b b a + = = = . Se obine 7, 0 70. a b ab = = =
P:283. Determinai cifra a astfel nct: 123456789 : ) ... ... ( = + + + + a a aa aaa aa a .
D.M. Btineu-Giurgiu, Bucureti i Neculai Stanciu, Buzu
Rezolvare. Observm c:
123456789 111111111 ... 111 11 1
9
999999999 ... 999 99 9
= + + + + =
+ + + +
. Aadar, 9 = a .
P:284. n liftul unei cldiri cu 10 etaje urc de la parter cinci persoane. n cte moduri pot cobor din
lift persoanele astfel nct pe un etaj s nu coboare mai mult de o persoan.
Prof. Adrian Stan, Buzu
Rezolvare: Din cele cinci persoane, prima are 10 posibiliti de coborre. La fiecare din acestea, a doua
persoan are 9 posibiliti de coborre. A treia persoan poate cobor n 8 moduri, etc. Cele cinci persoane
vor cobor n 10 9 8 7 6 3024 = moduri.

P:285. Se scriu numerele naturale n ordine cresctoare ncepnd cu 1. S se determine cifra de pe
poziia 2900. Prof. Adrian Stan, Buzu
Rezolvare:
Primele 999 de numere sunt scrise cu 9 + 290 + 3900 = 2889 cifre. Restul de cifre pn la 2900 este
de 2900 2889 = 11 cifre. Acestea evident, vor face parte din numere de patru cifre, care sunt n numr de
trei: 1000, 1001, 1002. A 11-a cifr din numerele de patru cifre este 0, prin urmare, a 2900-a cifr este 0.

P:286. Determinai toate numerele naturale care mprite la 1961 dau restul de 400 de ori mai mare
dect ctul.
Prof. Mircea Mario Stoica, Arad
Rezolvare:
Se folosete teorema mpririi cu rest, d=c+r, Cum 1961 r ( = rezult { } 0;1; 2;..........;1960 r e .
Din { } 400 0;1; 2; 3; 4 r c c = e .
n final, numerele cutate sunt { } 0; 2361; 4722; 7083; 9444 .
P:287. Aflai numrul care se mrete cu 198939, dac adugm, la dreapta lui, numrul 48.
Prof. Mircea Mario Stoica, Arad
Rezolvare: Fie (x) numrul care trebuie aflat (l scriem ntre paranteze, cci, nu tim cte cifre are);
(x)48 (x) 198939 = + (x) 100 48 (x) 198939 + = + (x) 99 198891 = (x) 2009 = .

- PROBLEME REZOLVATE -

Clasa a V-a

G:386. Determinai toate numerele naturale a pentru care exist exact 2014 numere naturale b care
verific relaia: 5 2 s s
b
a
.
Prof. D.M. Btineu-Giurgiu, Bucureti i Neculai Stanciu, Buzu
Rezolvare: (dat de Dl. Titu Zvonaru):
Avem
2 5 5
1
2
1
5 2
a
b
a
a
b
b
a
s s > > s s .
- Dac a este multiplu de 5, atunci b poate lua una dintre valorile
5
a
, 1
5
+
a
,...,
(

2
a
; adic b poate
lua 1
5 2
+
(

a a
valori;
- Dac a nu este multiplu de 5, atunci b poate lua una dintre valorile 1
5
+
(

a
, 2
5
+
(

a
,...,
(

2
a
; adic
b poate lua
(

5 2
a a
valori.
Avem de analizat 10 cazuri, n funcie de restul mpririi lui a la 10 ; fie N k e ; Sunt bune doar situaiile :
671 2014 1 2 5 10 = = + = k k k k a i 6710 = a ;
671 2014 2 1 5 2 10 = = + + = k k k k a i 6712 = a ;
671 2014 2 1 5 3 10 = = + + = k k k k a i 6713 = a ; n celelalte situaii, nu se obine k eZ
Deci { } 6713 , 6712 , 6710 e a .

G:387. Fie 1 2 3 4 ...... 49 B = i 24 14641 C = . Artai c C divide B.
Prof. Petre Punescu, Roiorii de Vede
Rezolvare: 14641 nu este divizibil cu 2, 3, 5, 7, dar este divizibil cu 11 deoarece
4
14641 11 . =
Cum C=(1 2 3 4 11 11 11 11)=(1 11) 2 11) (3 11) (4 11) rezult c C divide B.


G:388. Calculai :
{ { {
( ) } } } ( ) ( )
888
104 0
231 401 4
1 2 3 ... 200 : 201 10 :100000 19 : 3 1 101 102 103 2007 1997 100 ( + + + + +

.
Prof. Simion Marin , Rm. Srat
Rezolvare: tim c : 1+2+3+4+.+200=
200 201
100 201
2

=
100 201 :201=100=10
2
;(10
2
)
888
=10
1776
;10
1776
10
231 2007
10 = , 10
2007
:100000
401 2007
10 = :
(10
5 401 2
) 10 100 = = ; (100-19) :81-1=0 ; (2007-1997)
0
1 = ; 1 100 = 100 . Expresia dat este 100.

G:389. Artai c numrul
2013
100 = A se poate scrie ca o sum de dou ptrate perfecte.
Prof. Gheorghe Struu , Buzu
Rezolvare:
= = = = =
2 2 2013 2 2013 2 2013 2 2013
10 10 10 ) 10 ( 100 A
2 2012 2 2012 2 2 2 2012
) 10 8 ( ) 10 6 ( ) 8 6 ( ) 10 ( + = + = .

G:390. Aflai restul mpririi numrului
289
287 4017 +

la 2009.
Prof. Mircea Mario Stoica, Arad
Rezolvare: = + + = + + = + 2008 2009 287 82369 2008 2009 287 287 4017 287
287 287 2 289

+ + = 2008 ) 1 287 41 ( 2009
287
restul mpririi lui
289
287 4017 + la 2009 este 2008.


- PROBLEME REZOLVATE -
G:391. Artai c dac S este suma divizorilor naturali ai numrului 2010 atunci 34S este ptrat
perfect i 1734S este cub perfect. Prof. Nicolae Ivchescu, Craiova
Rezolvare: Cum 2010 2 3 5 67 = suma divizorilor lui 2010 este
1 1 1 1 1 1 1 1
2 1 3 1 5 1 67 1
3 4 6 68
2 1 3 1 5 1 67 1
S
+ + + +

= =

. Atunci,
2 3
) 17 3 2 ( 34 = S
i
( )
3
2
1734 2 3 17 S = .

G:392. 60 de muncitori, lucrnd cte 8 ore pe zi, au pavat un drum de lungime 900 m n 12 zile. Ci
muncitori pot pava n 10 zile, lucrnd cte 6 ore pe zi, un drum lung de 1200 metri?
Prof. Adrian Stan, Buzu
Rezolvare:
1) 60 muncitori ...............8h/zi..............12 zile ................900 m
2) x ? muncitori ..............6h/zi .............10 zile ...............1200 m
Din 1) rezult c un muncitor lucrez
900
60 12
m
zi

sau
900 5
60 12 8 6 8
m
h
=

.
Acelai muncitor dar n situaia de la 2) va lucra n 10 zile cte 6h/zi un numr de
8
50
10 6
8 6
5
=

m.
Aadar, la
50
8
m.1 muncitor, atunci 1200 m . x muncitori,
1200
192
50
8
x = = muncitori.
G:393. Aflai numrul natural xy , astfel nct
2
. 1573
xy yx
xy
x y
| |
+
=
|
+
\ .
.
Prof. Iuliana Trac, Scorniceti, Olt
Rezolvare:
( )
2
11
1573; 121 1573 13.
x y
xy xy xy
x y
+ (
= = =
(
+


G:394. a) Gsii trei numere naturale nenule i diferite care s verifice egalitatea 5a 11b 6c 0 + = ;
b) Artai c oricare ar fi numerele naturale nenule i diferite a, b, c, astfel nct, s aib loc
egalitatea 5a 11b 6c 0 + = , numrul A (a c)(b c) = se divide cu 55 .
Prof. Constantin Apostol, Rm. Srat
Rezolvare: a) Fie , *, m n m n e = N i fie 11 , 11 c m a n = = . Din 5 11n 11b 6 11m 0 + = . Se obine
b 5n 6m = + . Pentru m =2, n=3 rezult a=33, b=27, c=22.
b) Din egalitatea 5a 11b 6c 0 + = , obinem 5a 6c 11b + = sau 5a 6c 5b 6b + = + , de unde,
5(a b) 6(b c) = . Deducem c 5 divide b c i 6 divide a b .
Tot din egalitatea 5a 11b 6c 0 + = , adic din 5a 6c 11b + = , adunnd 6a n ambii membri, obinem
11a + 6c = 11b + 6a, de unde, 11(a b) = 6(a c). Rezult c 11 divide a c i 6 divide a b , ceea ce
tiam . Aadar, A se divide, i cu 11, i cu 5, deci se divide cu 55 .

G:395. Determinai numerele naturale x i y dac
5 3
7 39 2013 x y + = .
Prof. Gheorghe Drstaru, Buzu
Rezolvare: Cum
5
7x este un termen al sumei, rezult { } 3 , 2 , 1 287 2013 7
5 5
e s s x x x .
Singura soluie bun este 3 x = i 2 y = . { } (3; 2) S = .

G:396. Aflai toate numerele de forma abc tiind c abc =(a+b+c-9)(a+b+c-8)(a+b+c-7)
Prof. George-Florin erban, Brila
Rezolvare: Folosim faptul c produsul a trei numere consecutive e divizibil cu 6.
abc 3 , adic c b a + + este multiplu de 3; dar 9 a b c + + > { } ( ) 12,15,18, 21, 24, 27 a b c + + e .
Dac a+b+c =12 atunci 3 4 5 60 abc = = ceea ce e fals;
Dac a+b+c =15 atunci 6 7 8 336 abc = = , a=b=3 si c=6 , fals;

- PROBLEME REZOLVATE -

Dac a+b+c =18 atunci abc =9*10*11=990 , a=b=9 si c=0 ; n celelalte cazuri nu obinem soluii
deoarece abc =(a+b+c-9)(a+b+c-8)(a+b+c-7)(21-9)(21-8)(21-7)=2184. Numrul cutat este 990 .

G:397. Un numr natural de trei cifre mprit la rsturnatul su d ctul 3 i restul 175. Aflai
numrul. (n legtur cu problema S:E13.90 din SE al G.M.-B. martie 2013).
Titu Zvonaru, Comneti
Rezolvare: Fie abc numrul cutat , cu 0 = a . Deoarece 175 3 + = cba abc , deducem c 2 s c (dac
3 > c , atunci 175 3 + cba are mai mult de trei cifre), deci { } 2 , 1 , 0 e c
-dac 0 = c , atunci 175 3 + = cba abc b a 20 175 97 = ; trebuie ca 175 97 a s se termine cu cifra 0,
deci 5 = a . Avem 310 20 = b i nu obinem soluie;
-dac 1 = c , atunci 175 3 + = cba abc b a 20 474 97 = ; ca mai sus, 474 97 a se termin cu 0 --dac
a 97 se termin cu 4; obinem 2 = a i cum 474 2 97 < nu avem soluii nici n acest caz ;
-dac 2 = c , atunci 175 3 + = cba abc b a 20 773 97 = ; obinem 9, 5 a b = = 952 = abc .

Clasa a VI-a

G:398. Rezolvai n * * N N ecuaia
1
48 25 2009 10
y
x y

+ + = .
Prof. Mircea Mario Stoica, Arad
Rezolvare: Evident, { }
1 1
2009 10 0 10 2009 1 1, 2, 3, 4
y y
y

> s e . nlocuind pe y n relaia din
enun se obin soluiile ( ) { } , (940; 4), (1839;3), (1928; 2), (1936;1) x y e .

G:399. Aflai restul mpririi numrului
2013 2013 2013
2074 94 2046 x = + la 2013.
Prof. Gheorghe Drstaru, Buzu
Rezolvare: Restul este zero deoarece
2013 2013 2013
(2013 61) (61 33) (2013 33) x = + + + + =
2013 2013 2013 2013
2013 6133 2013 2013
61 61 33 33 M M M M

= + + + = .
Alt rezolvare dat de Dl. Titu Zvonaru:
Vom demonstra mai nti c dac a i b sunt prime ntre ele, atunci
n n n
b a b a + ) ( se divide cu ab .
Folosind descompunerea n factori, avem:
Mb Mb a b a M b a b a
n n n
= + = + ) ( ) ( ; Ma Ma b b a M a b b a
n n n
= + = + ) ( ) ( i atunci
Mab b a b a
n n n
= + ) ( .
Numrul dat se scrie = + + = ] 61 33 ) 33 61 [( 33 2046 61 2074
2013 2013 2013 2013 2013 2013 2013
x
2013 33 61 ) 33 2046 ( ) 61 2074 ( M M M M = + = , deci restul mpririi lui x la 2013 este zero.

G:400. Raportul a dou numere este egal cu
1
4
, iar suma lor este tot
1
4
.
a) Aflai cele dou numere ;
b) mprii numrul
1
4
n pri invers proporionale cu numerele determinate la punctul a)
c) mprii numrul
1
4
n pri direct proporionale cu numerele determinate la punctul a)
Prof. Constantin Apostol, Rm. Srat

Rezolvare: a) Fie a i b cele dou numere . Din datele problemei avem :
a 1
b 4
= i
1
a b
4
+ = . Din prima


- PROBLEME REZOLVATE -
egalitate deducem c b 4a = i, astfel, din a doua egalitate, obinem
1
a 4a
4
+ = , adic
1
5a
4
= , de unde,
1
a
20
= i, deci,
1
b
5
= ;
b) Fie x i y, cele dou pri ale numrului.
1
x y x y 1
4
20 5 20 5 25 100
+
= = = =
+
; deducem c
1
x
5
= i
1
y
20
= .
c) Fie u i v, cele dou pri ale lui
1
4
.
1
u v u v
4
1
1 1 1 1 5
20 5 20 5 20
+
= = = =
+
; deducem c
1
u
20
= i
1
v
5
= .

G:401. Artai c printre orice 305 numere naturale, dou cte dou coprime cuprinse ntre 2 i
2011 2010 exist cel puin un numr prim.
D.M. Btineu-Giurgiu, Bucureti i Neculai Stanciu, Buzu
Rezolvare: Presupunem prin reducere la absurd c toate cele 305 numere
305 2 1
,..., , n n n care, verific
ipoteza sunt compuse. Fie
i
p cel mai mic divizor prim al lui
i
n i p cel mai mare dintre
i
p .
Deoarece
305 2 1
,..., , n n n sunt coprime, factorii
305 2 1
,..., , p p p sunt distinci. Rezult 2011 > p (al 305-lea
numr prim este 2011). Deci, pentru acel n care are cel mai mic factor prim p avem
2011 2010 2011
2 2
> > > p n . Contradicie!

G:402. Fie numerele a, b, c, d, e, f astfel nct
15
13
a b c d e f
a b b c c d d e e f f a
+ + + + + =
+ + + + + +
,
aflai numrul
a b c d e f
m
a f a b b c c d d e e f
= + + + + +
+ + + + + +
.
Prof. Iuliana Trac, Scorniceti, Olt
Rezolvare: Notm
a b c d e f
n
a b b c c d d e e f f a
= + + + + +
+ + + + + +
, avem
a b b c c d d e e f
m n
a b a b b c b c c d c d d e d e e f e f
| |
| | | | | | | |
+ = + + + + + + + + + +
| | | | |
+ + + + + + + + + +
\ . \ . \ . \ .
\ .

15 63
6
13 15
f a
m m
f a f a
| |
+ + + = =
|
+ +
\ .
.

G:403. Fie triunghiul ABC cu 1, , BC AB a AC b = = = unde , * a beN . Artai c triunghiul ABC
este isoscel. Prof. Mariana Mrculescu, Craiova
Rezolvare: Din inegalitatea triunghiului 1 a b a b + ) > i 1 b a b a + ) > a b = , adic triunghiul
este isoscel.

G:404. n triunghiul ABC avem : D ( e BC) astfel
nct m( ) BAD X = 30
0
i m(
0
) 50 ADB = X ,
E ( ) AC e astfel nct m(
0
) 50 ABE = X . tiind c
{F} = ADBE i m(
0
) 20 ACB = X , artai
c m( XFCB) = 10
0
.

Prof. Simion Marin , Rm. Srat



- PROBLEME REZOLVATE

Rezolvare:

BFD este unghi exterior triunghiului ABF m(


0 0 0
) 30 50 80 BFD = + = X . n
BFD A calculm m(
0 0 0 0
) 180 (80 50 ) 50 FBD = + = X ABE EBD X X (BE este bisectoare
n ABC A . ADB este unghi exterior triunghiului ADC m( ) ADB = X m( DAC X ) + m( DCA X )
50
0
=m( DAC X ) +20
0
m(
0
) 30 DAC = X BAD DAC X X (AD este bisectoare n ABC A .
Bisectoarele (AD i (BE sunt concurente n F (CF este a treia bisectoare ; dar m(
0
) 20 ACB = X
0
( ) 10 m FCB = < .


Clasa a VII-a

G:405. S se scrie numrul 6033 2012 ca diferen de dou produse de cte trei factori, numere
naturale consecutive. Prof. Nicolae Ivchescu, Craiova
Rezolvare:
( )
2 2
6033 2012 3 2011 (2011 1) 2011 2011 3 2011 2 2011 1 = + = + + + =
2011 (2011 1)(2011 2) 2011(2011 1)(2011 1) 2011 2012 2013 2010 2011 2012 = + + + =

G:406. Artai c
4 4
2010 2010
32
1001 1009
| | | |
+ )
| |
\ . \ .
. Prof. Mircea Mario Stoica, Arad
Rezolvare: =
|
.
|

\
|
+
|
.
|

\
|
+ >
|
.
|

\
|
+ +
|
.
|

\
|
+ =
|
.
|

\
|
+
|
.
|

\
|
4 4 4 4 4 4
1009
1001
1
1001
1009
1 2
1009
1001
1
1001
1009
1
1009
2010
1001
2010

32 ) 2 2 ( 2
1001
1009
1009
1001
1 1 2
4
4
= + >
|
.
|

\
|
+ + + = , deoarece
1001 1009
2
1009 1001
+ ) .

G:407. Determinai xeN pentru care
2
4 9 7 . x x + + eN
Prof. Iuliana Trac, Scorniceti, Olt
Rezolvare: Observm c :
2 2 2
4 4 1 4 9 7 4 12 9 x x x x x x + + < + + < + + sau
2
2 1 4 9 7 2 3, x x x x + < + + < + x fiind natural
2
4 9 7 2 2 x x x + + = + ,adic
2 2
4 9 7 4 8 4 3. x x x x x + + = + + =


G:408. n primul an, din lotul de n semine al unei staiuni de cercetare agricol, nu a ncolit o
smn. n anul urmtor, din n 1 semine de acelai fel, nu a ncolit o smn. Situaia se repet x
ani, dup care experimentul nregistreaz ncolirea 100 %.
Aflai x, dac suma seminelor ncolite n intervalul respectiv este
x 1
3 n
2
+ | |

|
\ .
.
Prof. Ion Stnescu, Smeeni, Buzu
Rezolvare: Fie n-1 numrul de seminte ncolite n primul an, n-2, numrul de semine ncolite n al doilea,
n-3 , numrul de semine ncolite n al treilea an, ................., n-x, numrul de semine ncolite n al x-lea an.
Suma tuturor seminelor devine

x 1 x 1
S nx (1 2 3 x) x n 3 n
2 2
+ + | | | |
= + + + + = =
| |
\ . \ .
x = 3.

G:409. Aflai numerele naturale , x y i numrul prim p care verific relaia: 1
2
=
y
p x .
Neculai Stanciu, Buzu i Titu Zvonaru, Comneti
Rezolvare: Vom arta c ( ) { } ) 3 , 1 , 2 ( ); 2 , 3 , 3 ( , , e p y x . Evident 0 = x , i de asemenea 0 = y deoarece 2 nu
este ptrat perfect. Avem: ) 1 )( 1 ( 1
2
+ = = x x x p
y
, de unde rezult c exist numerele naturale
a i b astfel nct b a < < 0 cu
a
p x = 1 i
b
p x = +1 . Atunci 2 =
a b
p p sau 2 ) 1 ( =
a b a
p p .

- PROBLEME REZOLVATE -
A B
C
D
M
N
x
z
y x-y
x
3 z
x

3


avem dou posibiliti:
- ) 3 , 1 , 2 ( ) , , ( 3 , 1 , 0 3 , 0 2 1 , 1 = = = = = = = =

p y x p b a p a p p
b a b a
;
- ) 2 , 3 , 3 ( ) , , ( 2 , 2 , 1 2 2 , 1 , 2 1 1 , 2
1
= = = = = = = = =

p y x p b a a p p p
b a b a
.


G:410. Se consider dreptunghiul ABCD cu aria 3 (u.a.) i punctele M , respectiv N cu
) ( ), ( BC N AB M e e astfel nct aria triunghiului MND este 1 (u.a). Determinai minimul sumei
CN AM + .
D.M. Btineu-Giurgiu, Bucureti i Neculai Stanciu, Buzu

Rezolvare: Avem figura de mai jos.

Din relaiile: (1) | | | | 2 1 3 = = MND ABCD ;
(2) | | | | | | | | | | MND ABCD NCD MAD MBN = + + ,
rezult: 1 4
3 3
3 2
2 2
3
2
) )(
3
(
= = + + + = + +

yz xz
x
y
zy zx
x
y xz
x
y
y x z
x
.

Din inegalitatea mediilor avem c z y + este minim dac i numai
dac z y = .
Deoarece

=
=
z y
yz 1
, rezult 1 1 = = = z y . Obinem aadar
2 ) min( = + z y .



G:411. ntr-un triunghi ABC, M e(BC) i

0
m(BAM) 25 = , iar

0
m(CAM) 30 = , N (AC) e i

0
m(ABN) 45 = , iar

0
m(CBN) 55 = . Artai c latura (AB) este media geometric a segmentelor
(AM) i (BN) . Prof. Constantin Apostol, Rm. Srat

Rezolvare: Din datele problemei, rezult

0
m(ACB) 180 [m(BAC) m(ABC)] = +
0 0 0
180 (55 100 ) = + =
0
25 .
Deducem c ABM CBA A A ~ , avnd unghiul

ABC, comun i

BAM ACB . Scriind
proporionalitatea laturilor,
AB BM AM
BC AB AC
= = ;
AB AC
AM
BC

= (1)
Din

0
m(BAN) m(CBN) 55 = = , deducem c CBN CAB A A ~ , avnd unghiul

ACB, comun i

BAN CBN . Din proporionalitatea laturilor,
BC BN CN
AC AB BC
= =
AB BC
BN
AC

= (2)
nmulind relaiile (1) i (2), membru cu membru, obinem :
AB AC AB BC
AM BN
BC AC

= , de unde,
dup simplificri, rezult
2
AM BN AB = , relaie care arat c latura (AB) este media geometric a
segmentelor (AM) i (BN).


- PROBLEME REZOLVATE -

Clasa a VIII-a


G:412. Scriei numrul numrul
2013 2012
4 2011 + ca produs de doi factori.
D.M. Btineu-Giurgiu, Bucureti i Neculai Stanciu, Buzu
Rezolvare:
Observm c :
2013 2012
4 2011 + = . 4 ) 4 ( 4 ) ) 2011 (( 4 4 2011
4 4 4 503 4 503 2012 503 4
b a + = + = +


Dar, = + = + + = +
2 2 2 2 2 2 4 2 2 4 4 4
) 2 ( ) 2 ( 4 4 4 4 ab b a b a b b a a b a
= ) 2 2 )( 2 2 (
2 2 2 2
ab b a ab b a + + + , care reprezint identitatea Sophiei Germain.
Deci = +
2013 2012
4 2011 ) 2 2 )( 2 2 (
2 2 2 2
ab b a ab b a + + + , unde , 2011
503
= a
503
4 = b .

G:413. Determinai * neN pentru care este adevrat egalitatea:
( ) ( ) ( ) ( ) ( ) 2 3 2 3 2 3 ... 2 3 2 3
2 2 2 2 2 2 2 2
2 2 1 1
+ + + + +
n n n n
=
256 256
256 6561 .
Prof. Iuliana Trac, Scorniceti, Olt
Rezolvare: Amplificnd membrul din stnga al egalitii cu 1=3-2 i utiliznd produsul sumei cu diferena,
se obine
1 1
2 2
2 3
+ +

n n
. De asemenea, ( )
8 8
2
4 2
2 256 ; 3 3 81 6561 = = = = .
( )
11
8
3
2
2
2 256
3 3 6561 = = . ( )
11 8 3 8
8
2 2 2 2 8
2
8 256
2 2 2 2 256 = = = =

.
Am obinut: 10 11 1 2 3 2 3
11 11 1 1
2 2 2 2
= = + =
+ +
n n
n n
.

G:414. Se consider o funcie : , ( ( )) 2009 2008. f f f x x = 1 1 Artai c (1) 1 f = .
Prof. Doina i Mircea Mario Stoica, Arad
Rezolvare: Din ( ( )) 2009 2008 f f x x = (1), pentru x=1 se obine ( (1)) 1 f f = . (2). nlocuim n
(1) pe x cu f(1) se obine ( ( (1))) 2009 (1) 2008 f f f f = (3). Din (2) i (3) se obine
(1) 2009 (1) 2008 (1) 1 f f f = = .

G:415. Cercetai dac max
2
max( 5 7) x x + + aparine intervalului
1
1;
2
| (

(
\
.
Prof. Ion Stnescu, Smeeni,Buzu
Rezolvare:

2 2 2
5 25 25 53 5
5 7 ( 2 ) 7 ( )
2 4 4 4 2
x x x x x + + = + + + = . Rezult c maximul expresiei este
53
4
i
nu aparine
1
1;
2
| (

(
\
.

G:416. S se arate c pentru nici o valoare ntreag a lui n, raportul
3 2
3
n n n 2
n n 3
+ + +
+
, nu este numr
ntreg.
Prof. Constantin Apostol, Rm.Srat
Rezolvare: Observm c oricare ar fi a eZ, la numitor obinem numrul
3
a a 3 + , care se divide cu 3,
cci,
3
a a 3 + a(a 1)(a 1) 3 = + + , unde a(a 1)(a 1) + este un produs de trei numere ntregi
consecutive.
S artm c pentru nici un numr a eZ, numrul de la numrtor, nu se divide cu 3.
n raport cu mprirea la 3, un numr ntreg este de forma 3k, 3k 1 + sau 3k 2 + , k eZ.
Pentru a 3k = , obinem :
3 2
(3k) (3k) 3k 2 + + + =
3
2 M + ;
Pentru a 3k 1 = + , obinem :
3 2
(3k 1) (3k 1) (3k 1) 2 + + + + + + =
3
2 M +
Pentru a 3k 2 = + , obinem :
3 2
(3k 2) (3k 2) (3k 2) 2 + + + + + + =
3
1 M +

- PROBLEME REZOLVATE -

Dac numrtorul nu este divizibil cu un divizor al numitorului, deducem c numrtorul nu este
divizibil cu numitorul i, deci, raportul nu este numr ntreg.


G:417. n cubul ABCDEFGH de muchie a se consider M mijlocul lui | | AB . S se determine
distana de la H la planul (FMC).
Prof. Adrian Stan, Buzu
Rezolvare:
Fie { } N GB FC = . N este mijlocul lui
| | FC
i din triunghiul echilateral HFC rezult HN FC (1)

2
,
2
a
HG a GN NB = = = ,
2
a
MB = .
Deoarece 2
HG GN
NB MB
= = i

0
( ) ( ) 90 m G m B = = rezult HGN NBM A A ~

HNG NMB

0
( ) ( ) 90 m HNG m MNB + = (2). HN MN
Din (1) i (2) rezult ( ) HN FMC , iar ( )
2 2
6
, ( )
2
a
d H FMC HN HG GN = = + = .

G:418. Fie cubul ABCDABCD i ( ) ' M BB e . Precizai poziia lui M pe muchia ( ') BB dac

( )
4
cos ( ), ( )
5
m ADM ABC
(
=

. Prof. Gheorghe Drstaru, Buzu

Rezolvare: Deoarece AD MA i AD AB , unghiul dintre planele ) ( ADM i ) ( ABC este MAB < .
Avem:

( )
cos ( ), ( )
AB
m ADM ABC
AM
(
=


2 2
4
5
l
l BM
=
+

3
4
l
BM = .

G:419. Pe planul triunghiului dreptunghic ABC avnd cateta AB = 12 cm i ipotenuza
BC = 6 13 cm , se ridic perpendiculara BM = 10 cm . Calculai :
a) Distana de la M la AC ;
b) Distana de la M la centrul de greutate al triunghiului .
Prof. Simion Marin , Rm. Srat
Rezolvare: Deoarece MB ( ) ABC i AB AC rezult
MAAC (t . 3 ) ; deci MA = d(M , AC) . Din triunghiul
dreptunghic MBA obinem MA =
2 2
10 12 2 61 + = cm .
Din triunghiul ABC obinem
AC=
2 2
468 144 324 18 BC AB = = = cm.
Fie N mijlocul lui
| |
AC .
Rezult c AN= 9
2
AC
= cm. Din AABN calculm lungimea
medianei BN =15 cm . BG =
2
3
din BN =10 cm.
Din AMBG obinem MG = 10 2 cm .

- PROBLEME REZOLVATE -

Clasa a IX-a

L:268. S se determine , a be1astfel nct ecuaiile
2
(4 10) ( 8) 3 0 a x a x + + = i
2
( 8) (2 19) 9 0 b x b x + + + = s aib aceleai soluii. Prof. Adrian Stan, Buzu
Rezolvare:
Din condiia
4 10 8 1
8 (2 19) 3
a a
b b

= =
+ +
rezult 12 30 8 a b = + i 3 24 2 19 a b = . De aici,
prin rezolvarea sistemului
12 38
3 2 5
a b
a b
=

+ =

rezult 3 a = i 2 b = .

L:269. Determinai mulimea
2
3 3
x
A x
x x

= e e
`
+ +
)
1 Z . Prof. Constantin Dinu, Buzu
Rezolvare:
2
2
(3 1) 3 0
3 3
x
p px p x p
x x
= e + + =
+ +
Z . Pentru 0, p x = e 1 0 A >
2
3 6 1 0 p p + > { } 2; 1; 0 pe .
Pentru
3
2 2; ;
2
p x

= e
`
)
{ } 1 3; 1 ; p x = e { } 0 0 ; p x = e n concluzie,
3
3; 2; ; 1; 0 ;
2
A

=
`
)


L:270. Fie z y x , , numere reale, 0 , 0 = < x z , astfel nct xz y 2 < . S se demonstreze
inegalitatea: 0 3 ) ( ) (
2 2 2
< + + + + + + z z y x y z y x x .
Prof. Constantin Rusu, Rmnicu Srat
Rezolvare: Deoarece 0 > xz , avem 0 < x . Asociem numerelor reale z y x , , funcia : f 1 1 dat prin
z yt xt t f + + =
2
) ( . Dar 0 4 2
2
< < xz y xz y . Deducem c 0 ) ( < t f , R t e . Deci
0 ) ( , 0 ) ( , 0 ) ( < < < z f y f x f . Adunnd aceste trei inegaliti obinem cerina din enun.

L:271. Fie 0; .
2
x
t | |
e
|
\ .
Artai c
3 3
sin cos 1
5 12 13
x x
+ > . Prof. Ana Cismaru, Malu Mare, Dolj
Rezolvare:
13
1
) cos )(sin 12 3 (
1
cos 12 sin 3
) cos (sin
cos 12
cos
sin 3
sin
12
cos
5
sin
2 2 2 2
2 2 2 4 4 3 3
=
+ +
>
+
+
> + = +
x x
x x
x x
x
x
x
x x x
.

L:272. Artai c
2
1 1 1 1 1
......... , *
5 13 25 2 2 1 2
n
n n
+ + + + ( e
+ +
N .
Prof. Nicoleta Bran, Craiova
Rezolvare: Deoarece
2 2 2
2 2 1 ( 1) 2 ( 1) n n n n n n + + = + + ) +
2
1
1 2
1
1
1 1
2
1
) 1 ( 2
1
1 2 2
1
1 1 1
2
<
+
=
|
.
|

\
|
+
=
+
<
+ +
_ _ _
= = =
n
n
k k k k k k
n
k
n
k
n
k
.
L:273. tiind c x, y
*
+
e1 i x y < , s se compare numerele :
3
y
a
x
= i
2 2
b 2x xy y = + + .
Prof. Constantin Apostol, Rm. Srat
Rezolvare: Fie relaia dintre cele dou numere, care poate fi >, = sau < .


- PROBLEME REZOLVATE -

Aadar,
3
y
x

2 2
2x xy y + + x
3
y
3 2 2
2x x y xy + +
3 3
y x
3 2 2
x x y xy + +

2 2
(y x)(y xy x ) + +
2 2
x(x xy y ) + + y x x y 2x. Obinem cazurile :
I) Dac y 2x < , este < ;
II) Dac y 2x = , este = ;
III) Dac y 2x > , este > ;

L:274. S se arate c dac x, y, u, v e1 astfel nct
2 2
x + y =9 i
2 2
u + v =36 , atunci
| |
18;18 xu yv + e . Prof. Iuliana Trac, Scorniceti, Olt
Rezolvare:
Metoda 1: Alegem x=3cosa, y=3sina, u=6cosb , v=6sinb (care verific egalitile de mai sus)
Atunci, xu+yv= 18cos(a- b)
| |
18, 18 e .
Metoda 2: Conform inegalitii Cauchy-Buniakowski-Schwarz rezult
( ) ( )( )
2
2 2 2 2
xu+yv x +y u +v s =324, adic xu+yv
| |
18, 18 e .
L:275. Pentru * neN , se consider numerele n progresie aritmetic: 1 1 , n + +
2 2 1, n + + 1 1 n + + . S se determine raia progresiei.
Prof. Ionel Tudor,Clugreni,Giurgiu
Rezolvare: Numerele date sunt n progresie aritmetic dac i numai dac
(1 1) (1 1 )
2 2 1
2
n n
n
+ + + + +
+ + = 4 4 1 2 1 1 n n n + + = + + + + .
Notm 1 0 x n = + )
2
1 n x + = i
4
1 . n x + = Se obine, ecuaia
2 4
2 4x x x + = + de unde
4 2
4 2 0 x x x + = . Descompunnd, rezult
2 2 3 2
( 4) 2 0 ( 2)( 2 1) 0 x x x x x x + = + + = .
Pentru x>0 avem
3 2
2 1 1 0 x x + + ) ) . Rezult x=2 i atunci
4
1 16 1 256 255. n x n n + = = = = =
Am gsit progresia aritmetic 3,10,17 care are raia r = 7.

L:276. Fie : f 1 1astfel nct ( 1) ( ) 3 7 m f x n f x x + = + . Determinai , m ne1astfel nct
(2) 5, ( 3) 4 f f = = . Prof. Claudia Popa, Berca, Buzu
Rezolvare: Pentru 2 x = respectiv 3 x = rezult
4 5 1
23 28
, .
5 4 16 3 3
m n
n m
m n
=
= =

+ =



L:277. Dac |
.
|

\
|
e
2
, 0
t
k
x , n k , 1 = , atunci artai c
2
1 1
3
1
) 2 (sin n
x
tgx x
n
k k
k
n
k
k
> |
.
|

\
|
+
_ _
= =
.

D.M. Btineu Giurgiu, Bucureti i Neculai Stanciu, Buzu
Rezolvare: Este evident c : |
.
|

\
|
e > >
+
2
, 0 ,
2
2
3
2 sin t
x x
x
tg
tgx x
(1)
ntr-adevr, notnd ( ) 1 , 0
2
e = t
x
tg avem de demonstrat c:
( ) ( ) > + + e >

+
+
e >

+
+
) 1 ( 3 2 2 1 1 , 0 , 3
1
2
1
1
1 , 0 , 6
1
4
1
2
4 2 2
2 2 2 2
t t t t
t t
t t
t
t
t
t

) 1 , 0 ( , 0 3
2 4
e > + t t t , ceea ce este evident.
Prin urmare,

- PROBLEME REZOLVATE -
( ) n k x x tgx x
k
n
k
k
n
k
k k
, 1 ,
2
, 0 , 3 2 sin
1 1
=
|
.
|

\
|
e > +
_ _
= =
t
(2)
nmulind inegalitatea (2) cu 0
1
1
>
_
=
n
k k
x
i innd seama c:
2
1 1
1
n
x
x
n
k k
n
k
k
>
|
|
.
|

\
|
|
.
|

\
|
_ _
= =
, inegalitatea (2)
devine:
2
1 1 1 1
3
1
3
1
) 2 (sin n
x
x
x
tgx x
n
k k
n
k
k
n
k k
k
n
k
k
>
|
|
.
|

\
|
|
.
|

\
|
> |
.
|

\
|
+
_ _ _ _
= = = =
, ceea ce demonstreaz enunul.

L:278 S se demonstreze c ntr-un triunghi oarecare (cu notaiile obinuite) au loc relaiile:
a) GI
2
=
) ( 9
2
c b a + +
[(ab)(ac)(pa)+(ba)(bc)(pb)+(ca)(cb)(pc)].
b) GI
2
=
9
1
( p
2
+5r
2
16Rr). Prof. Marcel Chiri, Bucuresti
Rezolvare: Soluie. ) (
3
1
AC AB AG + = , AC
c b a
c
AB
c b a
b
AI
+ +
+
+ +
=
GI = AC
c b a
c
AB
c b a
b
)
3
1
( )
3
1
(
+ +
+
+ +
. Dar ) (
2
1
2 2 2
a c b AC AB + =
2
GI = GI GI = [ AC
c b a
c
AB
c b a
b
)
3
1
( )
3
1
(
+ +
+
+ +
][ AC
c b a
c
AB
c b a
b
)
3
1
( )
3
1
(
+ +
+
+ +
]
=
2
) ( 3
2
|
|
.
|

\
|
+ +
+
c b a
b c a
c
2
+
2
) ( 3
2
|
|
.
|

\
|
+ +
+
c b a
c b a
b
2
+
|
|
.
|

\
|
+ +
+
) ( 3
2
c b a
b c a
|
|
.
|

\
|
+ +
+
) ( 3
2
c b a
c b a
(b
2
+c
2
a
2
) =
=
2
) ( 9
1
c b a + +
[(a+c-2b)
2
c
2
+(a+b-2c)
2
b
2
+(a+c-2b) (a+b-2c) (b
2
+c
2
a
2
)]
=
) ( 9
1
c b a + +
(a
3
b
3
c
3
+2a
2
b +2a
2
c +2ab
2
+2b
2
c +2ac
2
+2bc
2
9abc).
Se verific uor c : = + + + + + + abc bc ac c b ab c a b a c b a 9 2 2 2 2 2 2
2 2 2 2 2 2 3 3 3

)] )( )( ( ) )( )( ( ) )( )( [( 2 c p b c a c b p c b a b a p c a b a + + =
innd cont de formulele: p =
2
c b a + +
, r =
p
c p b p a p
p
S ) )( )( (
= , S =
R
abc
4
avem
9
1
( p
2
+5r
2
16Rr) =
9
1
[ ]
8
) ( 4
) )( )( ( 5
4
) (
2
c b a
abc
c b a
c b a b a c a c b c b a
+ +

+ +
+ + +
+
+ +
=
) ( 9
1
c b a + +
[ ]
) ( 4
32 ) )( )( ( 5 ) (
2
c b a
abc c b a b a c a c b c b a
+ +
+ + + + + +
=
) ( 9
1
c b a + +
(a
3
b
3
c
3
+2a
2
b +2a
2
c +2ab
2
+2b
2
c +2ac
2
+2bc
2
9abc) = GI
2
.

L:279. Fie c b a , , numere reale pozitive. S se demonstreze inegalitatea:
2 2 2 2
) 9 ( ) ( 48 + + + s + + c b a c b a .
Prof. Constantin Rusu, Rmnicu Srat
Rezolvare: Cu inegalitatea mediilor avem:

6
3
2
1
3
1
3 3
2 2 2
2 2 2
2 2 2
+ + +
=
+
+ +
s
+ +
s
+ + c b a
c b a
c b a c b a
(1)


- PROBLEME REZOLVATE -


Dar,
+ + +
=
+
+ + +
s
+ + +
12
9
2
1
6
3
1
6
3
2 2 2
2 2 2
2 2 2
c b a
c b a
c b a


2
2 2 2 2 2 2
12
9
6
3
|
|
.
|

\
| + + +
s
+ + +

c b a c b a
(2)
Din (1) i (2) rezult concluzia. Egalitatea se obine dac c b a = = , 1
3
2 2 2
=
+ + c b a
, adic
1 = = = c b a .


Clasa a X-a

L:280. Fie
2
: , ( ) 14 42 f f x x x = + + 1 1 . Rezolvai n C ecuaia ( ( ( ( ( ))))) 0 f f f f f x = .
Prof. Dana Camelia, Didu Ileana, Craiova
Rezolvare:
2
2 2 2
( ) ( 7) 7 ( ( )) ( ( ) 7) 7 ( 7) 7 f x x f f x f x x = + = + = + . Analog,
3
2
( ( ( ))) ( 7) 7, f f f x x = + etc.
5
2
( ( ( ( ( ))))) ( 7) 7 f f f f f x x = +
5
2
( 7) 7 7(cos 0 sin 0) x i + = = +
32
2 2
7 cos sin , 0, 31
32 32
k
k k
x i k
t t | |
= + =
|
\ .
.

L:281. Fie , , 0 x y z) . Artai c
2 2 2
2 2 2
3 3 3
( 1) ( 1) ( 1)
3.
3 ( ) 1 3 ( ) 1 3 ( ) 1
x y z
yz xz xy
+ + +
+ + >
+ + +

n ce caz avem egalitate ?
Prof. Gabriel Tica, Bileti, Prof. Lucian Tuescu, Craiova
Rezolvare:
2 2 2
3
2
3
( 1) ( 1) ( 1)
3 3
( 1)( 1) ( 1)( 1)
3 ( ) 1
x x x
y z y z
yz
+ + +
> > =
+ + + +
+
_ _ [
. Egalitate pentru 1. x y z = = =
S-a aplicat inegalitatea mediilor:
2
3 3
3 ( ) 1 3 ( ) 1 1 ( 1)( 1) yz yz yz y z yz y z + = + s + + + = + + .

L:282. S se rezolve ecuaia
2 3
1
7
7
1 4
log 7 log log 0
3
x
x
x
+ = .
Prof. Adrian Stan, Buzu
Rezolvare: Din condiiile de existen rezult 0 x) i 1 x = .
7
7 7
7
1 1 1
: log 7 ; log log 2log
1
log
log 7
2
x
x
Avem x x
x x
= = = = .
3
1 7
7
1
log log
3
x x = .
Notnd
7
log x t = ecuaia dat devine
3 2
7 4 3 0 t t + + =
2
( 1)(7 7 3) 0 t t t + + =
1 2,3
1, t t = e1. Aadar, din
7
1
log 1
7
x x = = .

L:283. Folosind eventual identitatea
3
13 12 ( 1)( 3)( 4) x x x x x + = + , rezolvai ecuaiile:
a)
3
3 13 3 12 0;
x x
+ =
b)
2
lg lg( 25 ) lg( 1) lg(25 12) x x x x x + + = + + . Prof. Constantin Dinu, Buzu
Rezolvare: a) Cu notaia 0 3 > = y
x
, ecuaia devine
2
13 12 0 y y + = ( 1)( 3)( 4) 0 y y y + =
rezult 1, 3, 4 y y y = = = . De aici se obine c { } 0;1 ; xe

- PROBLEME REZOLVATE

b) Din condiiile de existen ale logaritmilor
2
0, 25 0, x x x ) + )
1 0, 25 12 0, x x + ) ) rezult
12
; ;
25
x

e +
`
)
Ecuaia devine
2
lg ( 25 ) lg( 1) (25 12) x x x x x + = +
3
13 12 0 x x + = { } 1;3 x e .

L:284. S se rezolve sistemul:

= + +
= + +
= + +
+
+
+
11 2 2 2
7 2 2 2
5 2 2 2
x z x z
z z y y
y y x x
. Prof. Struu Gheorghe, Buzu
Rezolvare: Notm: 0 2 ; 0 2 ; 0 2 > = > = > = c b a
z y x
. Astfel, sistemul devine:

= + +
= + +
= + +

= + +
= + +
= + +
12 ) 1 )( 1 (
8 ) 1 )( 1 (
6 ) 1 )( 1 (
11
7
5
c a
c b
b a
a ac c
c bc b
b ab a
3 log 3
0 1
1 2
2
= =
= =
= =
z c
y b
x a
.


L:285. S se rezolve n mulimea Z Z ecuaia: 7 3 100.
x y
= +
Prof. Ovidiu an, Rmnicu Srat
Rezolvare: Artm mai nti c ecuaia nu are soluii ntregi negative.
Dac 0, 0 7 (0,1), 3 (0,1) 3 100 (100,101)
x y y
x y < < e e + e , deci nu avem soluii.
Dac 0, 0 7 (0,1), 3 100 100
x y
x y < > e + > , deci nu exist soluii.
Dac 0, 0 7 1, 3 100 (100,101)
x y
x y > < > + e , dar nu exist xeZ cu 7 (100,101)
x
e deci din nou nu
avem soluii. Aadar 0, 0. x y > > Cazurile x=0 sau y=0 nu conduc nici ele la soluii.
Pentru 0, 0 x y > > scriem ecuaia sub forma:
3 3 5 5
7 343 3 243 7 (7 1) 3 (3 1).
x y x y
= =
Cum
3 5
(7 , 3 ) 1 = i observnd c ecuaia nu are soluii 3, 5 x y < < rezult
3 5
7 / (3 1).
y

Dac
5
5, 3 1 1(mod3)
y
y y

> e N . Dar
3
7 343 1(mod3) = deci 1 1(mod3) , fals.
Rmne deci 5 y = i apoi 3. x = n concluzie, ecuaia admite soluie unic,
( ) { }
3;5 S = .

L:286. Fie , 2 n n e > N i numerele reale 0 a b ) ) pentru care ( )
n
a b a b = + .
S se arate c: a)
1 2 2
;
n n
a b a b
+
= + b)
2 1
1 2 2
( )
n
n
a b
a b
+
+

este ptratul unui binom.


Prof. Ionel Tudor, Clugreni, Giurgiu
Rezolvare:
Mai nti facem observaia c exist numerele a>b>0 i n care verific condiiile din ipotez, de
exemplu a=3,b=1 i n=2.
( ) ( ) 0
n
n n
n
a b a b a b a b a b = + + = = ) .
Atunci ( )( ) ( ) ( )
1 2 2
1 1
n
n n
n n
a b a b a b a b a b a b a b
+
+ +
= + = = = + .

2 1 2 1 2 1
2
2 2
1 2 2
( ) ( ) ( )
( ) ( ) ( )
n n n
n n
n n
a b a b a b
a b a b a b
a b a b
a b
+ + +
+

( = = = = = +

+

.

L:287. Rezolvai ecuaia
ln ( 1) 3 3 2
2
3 ln ( 1) 3 2
x x
x x

=

.
Prof. Petre Punescu, Roiorii de Vede


- PROBLEME REZOLVATE

Rezolvare: Se face notaia ln ( 1) 0 x x a = ) i 3 2 b = , atunci
3
2
3
b a
b a

1
2 y
y
+ = unde s-a fcut notaia
3
b
y
a
=

. Se obine y =1 dup care


2 2
2 ln ( 1) 2 a x x x x e = = = cu soluiile
2
1,2
1 1 4
2
e
x
+
= care respect condiiile de
existen x
2
-x>o , ln(x
2
-x)>0 i numitorul diferit de zero.

L:288. Se consider mulimile ] , (
1
o = A i ) , [
2
+ = | A , unde 36 4 9
2 2 2 2
+ + + = y x y x o ,
2
) 6 ( + = xy | , iar y x, numere reale. S se arate c:
1) a) =
2 1
A A mulime vid, R y x e , ; b)Precizai relaia dintre x i y pentru care | o = ;
2) Aflai valorile lui x i y pentru care ] 50 , 49 [
2 1
= A A .
Prof. Constantin Rusu, Rmnicu Srat
Rezolvare: 1) Demonstrm c | o > . n adevr, o se scrie succesiv astfel
) 3 )( 2 ( ) 9 ( 4 ) 9 ( 36 4 9
2 2 2 2 2 2 2 2 2 2 2
+ + = + + + = + + + = y x y y x y x y x o . Avem de artat c
2 2 2 2 2
) 3 2 ( ) 3 )( 2 ( + > + + y x y x , ceea ce este adevrat conform inegalitii C-B-S.
Sau 0 ) 2 3 (
2
> > y x | o . Egalitatea se obine dac
3
2
=
y
x
. Pentru punctual 2) avem de rezolvat
sistemele:

= + + +
= +
50 36 4 9
7 6
:
2 2 2 2
1
y x y x
xy
S i

= + + +
= +
50 36 4 9
7 6
:
2 2 2 2
2
y x y x
xy
S .
Sistemul
2
S nu are soluii reale deoarece a doua ecuaie devine 0 169 36 50 4 9
2 2
< = + y x . Se obin
soluiile:
)
`

|
.
|

\
|

|
.
|

\
|
=
2
3
,
3
2
,
2
3
,
3
2
), 1 , 1 ( ), 1 , 1 ( S .


L:289. S se rezolve n 1 1 1 sistemul de ecuaii:

2 3 1 3 4 1 4 2 1 30
2 1 3 1 4 1 15
3 3 3 3
x y y z z x
x y z
+ + + + + +

+ + + + + =

+ + =

. Prof. Marcel Chiri, Bucureti


Rezolvare: Ridicnd la ptrat prima ecuaie i innd cont de inegalitatea mediilor obinem:
(2x+1) + (3y+1) + (4z+1) = 225 2 (2 1)(3 1) x y + + 2 (3 1)(4 1) y z + + 2 (4 1)(2 1) z x + +
2x + 3y + 4z + 3 > 225 ( 2x+1 + 3y + 1 ) ( 3y + 1 + 4z +1 ) ( 4z + 1 + 2x + 1)
6x + 9y + 12z 216 2x + 3y + 4z 72.
n ecuaia a doua innd cont de inegalitatea mediilor i de inegalitatea precedent obinem:
3 2 3 1 2 3 1 3 4 1 4 2 1 3 3 30 3 4 1 4 2 1 2 3 4 15 87 30
3 3 3 3 3 3 3 3 3 3 3
x y x y y z z x y z z x x y z + + + + + + + + + + + + + + + + +
= + + > = =
Se obine egalitate atunci cnd 2 1 3 1 4 1 x y z + = + = + .iinnd cont de prima ecuaie obinem
x=12, y=8 i z=6.

Clasa a XI-a

L:290. ntr-o progresie aritmetic ( )
1
n
n
a
>
, suma primilor n termeni este
2
4 8
3
n
n n
S
+
= . S se
calculeze
1 2
...
lim
n
n
n
a a a
n a

+ + +

. Prof. Adrian Stan, Buzu




- PROBLEME REZOLVATE -



Rezolvare: Cum
1
8 4
, 1,
3
n n n
n
a S S n

+
= = > iar
2
1 2
4 8
... ,
3
n n
n n
a a a S
+
+ + + = = rezult,
2
2
1 2
2
4 8
... 4 8 1
3
lim lim lim .
8 4
8 4 2
3
n
n n n
n
n n
a a a n n
n
n a n n
n

+
+ + + +
= = =
+
+


L:291. Fie
1 1 2 2 1
( ) , 3, 2, , 1.
n n n n n
x x x x x x n
> + +
= = = + >
a) Artai c
1
2 , 1;
n
x n
n
s + >
b) irul
1
( )
n n
x
>
este convergent i lim 2;
n
n
x

=
Prof. Ion Nedelcu, Ploieti, Prof. Lucian Tuescu, Craiova
Rezolvare: a) Se arat prin inducie.
2
1 1 1
2 2 2
1 2
n
x
n n n
+
s + + + s +
+ +
3 2
2 2
2 1 4 9
2 4 3 4 0, 2.
4 4
n n
n n n n
n n n n
+ +
s + > >
+ + +

b) Se arat prin inducie c 2
n
x > .
1
2; 2 , 1; lim 2
n n
n
x n x
n

(
e + > =
(

.

L:292. Se consider dezvoltarea
10 2 10
0 1 2 10
( 3) ... . X a a X a X a X = + + + + S se calculeze
1 2 3 10
2 3 ... 10 . a a a a + + + + Prof. Florentina Popescu, Buzu
Rezolvare: Derivnd relaia din enun, se obine
9 2 9
1 2 3 10
10( 3) 2 3 ... 10 X a a X a X a X = + + + + i
pentru 1 X = , rezult
9
1 2 3 10
2 3 ... 10 10 2 a a a a + + + + = .

L:293. Se consider funcia : f D _ 1 1,
3 3
( ) .
3 3
x
f x
x
+
=


a) S se determine domeniul maxim de definiie a funciei i s se studieze continuitatea lui f pe aceast
mulime.
b) Calculai ( )
2
3
3
lim ( )
x
x
f x

. Prof. Constantin Dinu, Buzu


Rezolvare:
a) ( ) 3 3 0 0; 6 x D ) = ; Dup explicitarea modulului, rezult
( |
( )
6
, 0;3
( ) ,
, 3; 6
6
x
x
x
f x
x
x
x


Deoarece
3 3
3 3
6
lim lim (3) 1
6
x x
x x
x x
f
x x

( )

= = =

, atunci f este continu pe D.


b)
1
2
3
3
3
3
6
lim ,
x
x
x
x
e
x

(
| |
=
|
\ .
1
2
3
3
3
3
lim
6
x
x
x
x
e
x

)
| |
=
|

\ .
( )
2
2
3
3
3
lim ( ) .
x
x
f x e

=


L:294. Fie * neN i , ( )
n
A B M e C inversabil. Artai c:
a) Dac
1 1
n
AB I BA

= + atunci A B este inversabil.
b) Dac
1 1
n
BA I AB

= + atunci A B + este inversabil.
Prof. Otilia Drgan, Prof. Ovidiu Cioponea, Craiova


- PROBLEME REZOLVATE -

Rezolvare:
a) Din
1 1 1 1
( )( )
n n n
A B A B I BA AB I I

= + = det( ) 0 A B A B = este inversabil.
b) Din
1 1 1 1
( )( )
n n n
A B A B I BA AB I I

+ = + = det( ) 0 A B A B + = + este inversabil.

L:295. Fie , , 0 x y z) cu 1. x y z + + = Artai c:
a)
2
( ) ( ) ( ) ;
3
z x y
x y y z z x + + + s
b)
1
;
9
y z z x x y
x y z
+ + +
s Prof. Ion Nedelcu, Ploieti, Prof. Lucian Tuescu, Craiova
Rezolvare:
a) Fie : (0;1) , ( ) ln(1 ) f f x x x = 1 , atunci,
2
2
''( ) 0, (0;1)
( 1)
x
f x x
x
+
= ( e

f este concav pe
(0;1) de unde rezult conform inegalitii
( ) ( ) ( )
3 3
x y z f x f y f z
f
+ + + + | |
>
|
\ .
ceea ce trebuia artat.
b) Fie : (0;1) , ( ) (1 ) ln g g x x x = 1 , atunci,
2
1
''( ) 0, (0;1)
x
g x x
x
+
= ( e g este concav pe
(0;1) de unde rezult conform inegalitii
( ) ( ) ( )
3 3
x y z g x g y g z
g
+ + + + | |
>
|
\ .
ceea ce trebuia artat.


L:296. Fie
| |
: ; , 0 f a b a b ( ( 1 de dou ori derivabil i
( )
( ) ( ) ( ). a f b b f a a b f ab + = +
Artai c exist ( ) ; c a b e astfel nct ''( ) 0. f c =
Prof. Liviu Smarandache, Ramona Puchiu, Craiova
Rezolvare:
( ) 1
; c a ab - e astfel nct
1
( ) ( )
'( )
( )
f ab f a
f c
a b a

i
( ) 2
; c ab b - e astfel nct
2
( ) ( )
'( )
( )
f b f ab
f c
b b a

. Din enun i ( ) ( )
1 2 1 2 1 2
'( ) '( ) , ; c c f c f c c c c a b ( = - e c astfel nct
''( ) 0 f c = .

L:297. S se rezolve n 1 1sistemul
2 3
2 3
x y
y x
x
y
+ =

+ =

. Prof. Aurel Chiri, Slatina


Rezolvare:
Pentru x y ( 3 2 2 3
y x y x
x y y x = + ( + ( ( , fals. Analog pentru . y x ( Atunci, x y =
2 3
x x
x + = . Fie
| |
: 2;3 , ( )
x
f f t t = 1 . Conform teoremei lui Lagrange, ( ) 2; 3 c - e astfel nct
{ }
1 1
3 2 0;1
x x x x
x c x x c x

= = e { } ( ; ) (0; 0), (1;1) x y e .

L:298. Fie curba de ecuaie
2
2 2, 0. x y x o o = ) Prin punctul de inflexiune se duce o
perpendicular pe o dreapt variabil ce trece prin origine. S se afle locul geometric al interseciei.
Prof. Claudia Popa, Berca, Buzu
Rezolvare:
2 3 4
2 2 2 4 4 12
( ) '( ) ''( )
x x x
y f x f x f x
x x x o o o
+
= = = = . Din
0
''( ) 0 3 f x x = = . Punctul
de inflexiune este
4
3;
9
I
o
| |
|
\ .
iar ecuaia locului geometric este ecuaia cercului

- PROBLEME PROPUSE -


2 2
2
2
3 2 9 4
2 9 4 81
x y
o o
| | | |
+ = +
| |
\ . \ .
de centru
3 2
;
2 9
C
o
| |
|
\ .
i raza
2
9 4
4 81
R
o
= + .


Clasa a XII-a

L:299. Fie
2 2
: ( ) ( ) f M M Z Z ,
1 3
( )
1 4
a
f x X
a
+ | |
=
|
+
\ .
. S se determine aeZ astfel nct f s
fie automorfism al grupului ( )
2
( ), M + Z .
Prof. Adrian Stan, Buzu
Rezolvare:
Fie f automorfism al grupului ( )
2
( ), M + Z , rezult f e bijectiv
1 3
1 4
a
a
+ | |

|
+
\ .
e inversabil, deci,
( 1)( 4) 3 0. a a + + + =
1
2
1 3 4 3
1
1 4 1 1 5 7
a a
a a a a

+ + | | | |
=
| |
+ + + +
\ . \ .
4, 3, 1, 1 a a + + trebuie s fie divizibile cu
2
5 7 a a + + adic
2
5 7 1 a a + + = sau 1 .
Din { }
2
5 7 1 2; 3 a a a + + = e i din
2
5 7 1 a a a + + = eZ.

L:300. Se consider funciile ( ) , : 1; , f F 1
2
1 1 1
( ) ln(1 )
1
f x
x x x
| |
=
|
+
\ .
, iar
1 1
( ) ln(1 ) F x
x x
= .
a) Demonstrai c F(x) este o primitiv a lui f(x);
b) Calculai
2
( ) ( )
e
f x F x dx
)
. Prof. Constantin Dinu, Buzu
Rezolvare: a) F este o primitiv a lui f deoarece
, ,
,
1 1 1 1
) ln(1 ) ln(1 ) ( ). F x f x
x x x x
| | (
= + =
|
(
\ .

b)
2
2
2
1 1 1 1
( ) ( ) ( ) ln(1 )
2 2 2 2
e
e e
f x F x dx F x
x x
(
= = =
(

)
2
2
1 1 1 1 1
ln (ln )
2 2 2
e
e e
(
| |
= =
(
|
\ .
(


( )
2
2
2
1 1 1
ln( 1) 1 ln 2 .
2 2
e
e
(
=
(



L:301. S se calculeze:
)
+ + + + +
+
= dx
a x x x x
x
a I
2 3 4 2
1 2
) (
2 3 4
, ae1 .
Prof. Constantin Rusu, Rmnicu Srat
Rezolvare: ) (a I se mai scrie
)
+ + + + +
' +
dx
a x x x x
x x
2 3 4 2
) (
2 3 4
2
, ceea ce sugereaz scrierea numitorului
dup puterile lui x x +
2
. Astfel:
= + + + + + a x x x x 2 3 4 2
2 3 4
| o + + + + ) ( ) (
2 2 2
x x x x , iar prin identificarea coeficienilor gsim
a + = = = 2 , 3 , 1 | o . Cu substituia t x x = +
2
, avem
)
+ + +
=
a t t
dt
a I
2 3
) (
2
.




- PROBLEME REZOLVATE -


Dac
4
1
0 4 8 9 < > = A a a , atunci =
|
|
.
|

\
|

|
.
|

\
|
+ = + +
|
.
|

\
|
+ = + + +
2
2 2
2
4
1
2
3
2
4
9
2
3
2 3 a t a t a t t
|
|
.
|

\
|
+ +
|
|
.
|

\
|
+ = a t a t
4
1
2
3
4
1
2
3
i = +
+ +
+

= C
a t
a t
a
a I
4 1 3 2
4 1 3 2
ln
4 1
1
) (
C
a x x
a x x
a
+
+ + +
+ +

=
4 1 3 2 2
4 1 3 ) ( 2
ln
4 1
1
2
2
.
Dac
4
1
0 4 1 = = = A a a , atunci: C
x x
C
t
t
dt
a I +
+ +
= +
+
=
|
.
|

\
|
+
=
)
3 2 2
2
2
3
1
2
3
) (
2 2
.
Dac
4
1
0 > < A a , atunci = +

=
|
|
.
|

\
|
+ |
.
|

\
|
+
=
)
C
a
t
arctg
a
a t
dt
a I
4
1
2
3
1 4
2
4
1
2
3
) (
2
2


C
a
x x
arctg
a
+

+ +

=
1 4
3 2 2
1 4
2
2
.

L:302. S se calculeze: dx
x
x a
)

+
+
1
1
2
1
arccos
, unde 0 > a .
D.M. Btineu Giurgiu, Bucureti i Neculai Stanciu, Buzu
Rezolvare: Fie dx
x
x a
I
)

+
+
=
1
1
2
1
arccos
, n care facem schimbarea de variabil t t u x = = ) ( , cu
1 ) ( = ' t u , 1 ) 1 ( = u , 1 ) 1 ( = u i obinem:
( ) I dt
t
a dt
t
t a
dt
t
t a
I
+
+ =
+
+
=
+
+
=
) ) )

1
1
2
1
1
2
1
1
2
1
1
1
arccos
) 1 (
1
) arccos(
t
t
.
Deci,
2
) ( ) ( 2
1
1
t
t t + = + =

a arctgt a I , de unde
( )
4
a
I
+
=
t t
.


L:303. Aflai primitivele funciei
2011
cos 2
: , ( )
(sin cos 2010)
x
f f x
x x
=
+ +
1 1 .
Prof. Liviu Smarandache, Ivnescu Ionu, Craiova
Rezolvare: Se face substituia sin cos (cos sin ) x x t x x dx dt + = =
2011
cos2
(sin cos 2010)
x
dx
x x
=
+ +
)

2011 2011 2010 2011
(cos sin )(cos sin )
2010
(sin cos 2010) ( 2010) ( 2010) ( 2010)
x x x x tdt dt dt
dx
x x t t t
+
= = =
+ + + + +
) ) ) )

2009 2010 2010
1 1 1 sin cos 1
.
2009 ( 2010) ( 2010) 2009(sin cos 2010)
x x
C
t t x x
+ +
= + = +
+ + + +




- PROBLEME REZOLVATE -

L:304. S se calculeze integrala
2 3 2
2 3
0
4 6 8 3
( 1)
x x x
dx
x x
+ + +
+ +
)
. Prof. Constantin Dinu, Buzu
Rezolvare:
2 2 2 3 2 2
2 3 2 3 2 3
0 0 0
4 6 8 3 2( 1)(2 1) 2 1
( 1) ( 1) ( 1)
x x x x x x x
dx dx dx
x x x x x x
+ + + + + + +
= + =
+ + + + + +
) ) ) 2 2 2
2 2
1 1 108
2
0 0 1 2( 1) 49 x x x x
=
+ + + +
.

L:305. Fie a
+
e1 . S se arate c
2
lg( 9 1 3 ) 0
a
a
x x dx

+ + =
)
. Prof. Adrian Stan, Buzu
Rezolvare: Fie
2
: , ( ) lg( 9 1 3 ) f f x x x = + + 1 1 . Se arat c f este impar:
2 2
2
1
( ) lg( 9 1 3 ) lg lg1 lg( 9 1 3 ) ( )
9 1 3
f x x x x x f x
x x
= + = = + + =
+ +
.

L:306. S se rezolve n 1 ecuaia:
6
8 3 1 0 x x x = .
Prof. Ionel Tudor,Clugreni,Giurgiu
Rezolvare: Impunem condiia 0 x > . Valoarea x=0 nu verific ecuaia.
Pentru x>0 notm 0 y x x = ) i ecuaia devine succesiv:
4
8 3 1 0 y y =
4 3 2 3 2 2
8 4 2 4 2 4 2 1 0 y y y y y y y y + + =
2 2 2 2
2 (4 2 1) (4 2 1) (4 2 1) 0 y y y y y y y y + + =
2 2
(4 2 1)(2 1) 0 y y y y + + = Pentru ecuaia
2
1,2
1 5
4 2 1 0 20
4
y y y

= A = = . Convine
doar
1 5
4
y
+
= . Avem
3
3 2
3 5 3 5
0,
8 2
x y x
+ +
= = = ) de unde
3
3 5
.
2
x
+
=
Pentru ecuaia
2
2 1 0 7 y y + + = A = ecuaia nu are soluii reale.
Deci , n 1 ,ecuaia dat are doar soluia
3
3 5
.
2
x
+
=

tiai c .....?

Pe 28 mai 585 .e.n este observat o eclips de soare i datat cu exactitate pentru prima dat,
eveniment ce avea s rmn n istoriografie ca fiind momentul n care conductorul mezilor
Cyaxares trebuie s ntrerup lupta cu otile lidianului Alyattes deoarece ziua devenise noapte.
Evenimentul a fost prevestit de Thales din Milet care nc din secolul VI .e.n calculeaz i viitoarea
eclips din 14 martie 190 .e.n.

Cometa Halley ce poart numele astronomului englez Edmund Halley (1656 1742) este
observat i consemnat pentru prima dat de ctre chinezi acum circa 239 .e.n. Micarea sa
periodic n jurul Soarelui devine vizibil n jurul Pmntului la circa 77 de ani i de obicei oamenii de
rnd considerau c aceasta prevestea ceva ru ca i n 160 .e.n odat cu rzvrtirea iudeilor
mpotriva dominaiei seleucide sau n 1066 n btlia de la Hastings a lui William Cuceritorul (1027
1087)n care acesta a crezut c e de bun augur pentru el . Ultima dat, cometa a fost vizibil n 1986 i
va mai fi vizibil urmtoarea dat n 2061.

Construirea observatorului astronomic de la Greenwich ncepe n 1675 la iniiativa regelui
Charles al II-lea (1630 1685) pentru a veni n sprijinul navigatorilor i pentru a defini meridianul
zero care este marcat printr-o band de alam ce trece prin curtea institutului. Vizitatorii care calc
pragul institului pot vedea acest meridian i chiar pot clca pe el.


- PROBLEME PROPUSE -
Este o problem simpl s faci lucrurile s fie complicate,
dar e foarte complicat s le faci simple.
Legea Lui Meyer


4. Probleme propuse


nvmnt primar

P:288. Priviti cu atenie urmtorul tablou cu numere:
12 46 58 62
2 24 40 ?
Descoperii regula dup care sunt aezate numerele i completai spaiul lipsa.
Prof. Gabriela Stoenescu si Florin Stanescu, Dmbovia
P:289. Suma a trei numere este 19. Dac primul numr adunat cu triplul celui de-al doilea este 19 , iar al
doilea adunat cu al treilea este 15 , s se afle numerele .
Elev. Alexia Dragan, Bucureti
P:290. Suma a cinci numere naturale este 416 . Aflai numerele tiind c primele patru numere sunt impare
consecutive , iar al cincilea este o treime din suma celorlalte .
Prof. Marcela Marin, Rm. Srat

P:291. Un elev a citit o carte n 3 zile. n prima zi a citit jumtate din numrul paginilor, n a doua zi un
sfert din rest, iar n a treia zi 24 de pagini. Cte pagini are cartea?
Prof. Anton Maria, Berca
P:292. Dac mprim diferena numerelor 528 i 179 la suma dintre 7 i un numr necunoscut, obinem
ctul 7 i restul 6. Afl numrul necunoscut.
Prof. Lupan Cristian-Cosmin , Buzu

P:293. Aflai numrul necunoscut din {[(x + 3) x 3 + 3] x 3 + 3} x 3 + 3 = 309
nv. Lupan Ion, Plecoi, Berca

P:294. Suma a patru numere este 700. S se afle numerele tiind c suma dintre primul i al treilea este de
dou ori mai mare dect al doilea, iar al patrulea este jumtate din al doilea i cu 6 mai mare dect primul.
Prof. Lupan Nicoleta-Gabriela, Berca

P:295. Diana i mama ei au mpreun 30 de ani. Peste 6 ani, Diana va avea o cincime din vrsta mamei
sale. Ci ani are fiecare acum?
Prof. Marinescu Gabriela, Vadu Paii, Buzu

P:296. ntr-un scule sunt 20 de baloane galbene i roii. Dac scot 10 baloane fr s le vd, sigur este
unul rou, iar dac scot 12 baloane, sigur unul este galben. Cte baloane sunt de fiecare culoare n scule?
Prof. Ticea Daniela, Buzu

P:297. Diferena dintre vrsta corbului mare i vrsta corbului mic este de 51 de ani, iar o jumtate din
vrsta corbului mare este de dou ori mai mare dect vrsta corbului mic.
Ct mai are de trit fiecare, tiind c un corb, dac ar tri 10 ani i nc 20, ar tri a zecea parte dintr-un
mileniu?
Prof. Vrabie Marioara , Berca



- PROBLEME PROPUSE -


Clasa a V-a
G:420. Se d numrul ( ) | | { }
5: 2 3 1 14 :15 3: 1 2 (45:15 2) : 2 2008 A = + + + + (

.
S se calculeze suma divizorilor numrului A.
Prof. Nicolae Ivchescu, Craiova

G:421. Determinai cele mai mici numere naturale abcde , respectiv fghij astfel nct: fghij abcde = 2
i cele dou numere mpreun s utilizeze toate cifrele de la 0 la 9.
D.M. Btineu-Giurgiu, Bucureti i Neculai Stanciu, Buzu
G:422. Aflai numrul natural n, astfel nct 2013 4024 2 2013 4024 ... 2013 4024 n + + + s fie
ptrat perfect. Prof. Ion Stnescu, Smeeni, Buzu
G:423. a) S se arate c fracia
544
225
F = este ireductibil.
b) Determinai neNcare verific egalitatea
5 3 544
.
3 5 225
n n
| | | |
=
| |
\ . \ .

Prof. Ionel Tudor, Viorica Dogaru, Clugreni, Giurgiu
G:424. Determinai numerele naturale a i b astfel nct fracia
1961
7 3 21 ab a b +
s fie echiunitar.
Prof. Mircea Mario Stoica, Arad
G:425. S se gaseasc numerele naturale m i n tiind c : 1
n
+2
n
+.....+2013
n
=45
m
-12 .
Prof. erban George-Florin, Brila
G:426. Determinai perechile de numere naturale a, b , pentru care 2ab 3a b 21 .
Prof. Constantin Apostol, Rm. Srat
G:427. Determinai numerele
__
ab , i trei numere naturale consecutive cu primul numr prim, tiind c prin
mprirea lui
__
ab la cele trei numere numere, se obin ca resturi precedentele celor trei numere.
Prof. Gheorghe Ghi, Buzu
G:428. Mulimile A i B sunt astfel nct ( ) 140 card A B = , ( ) 50 card A B = i ( ) 60 card B A = .
S se calculeze ( ). card A B Prof. Adrian Stan, Buzu
G:429. Prin mprirea numrului a la numrul b
( )
*
, a beN obinem ctul 7 i restul 12.
Artai c 70 490 111 a b este ptrat perfect i cub perfect. Prof. Iuliana Trac, Olt
G:430. Simplificai fracia:
9
1 1 1 ... 1 ...
9
cifre
a aa aaa aaa a + + + +
_
. Prof. Sorina Vcrean, Cluj-Napoca



Clasa a VI-a
G:431. Numerele 0 x y ) ) i * neN verific egalitatea
2 1 2 1
.
n n
x y x y
+ +
= + Artai c numrul
2 2 n n
x y
este supraunitar. Prof. Ionel Tudor, Clugreni, Giurgiu
G:432. S se arate c pentru orice numr natural n numrul
1 2
2011 2013 2015
n n n
N
+ +
= + + nu poate fi
ptrat perfect. Prof. Gheorghe Ghi, Buzu
G:433. Determinai numerele naturale x,y,z tiind c
12
9 18
x y
z
= = .
Prof. Doina i Mircea Mario Stoica, Arad

- PROBLEME PROPUSE -

G:434. Aflai toate numerele de forma abca care verific simultan condiiile :
a) abca . 11
b) Restul mpririi numrului abca la numrul ab este un numr cu trei divizori .
c) b <a .
Prof. erban George-Florin , Brila
G:435. Dac
1 2 2010
, ,..., * x x x e1 i
2 3 2010 1 1 2
1 2 2 3 2010 1
....
2010 2010 2010
x x x x x x
x x x x x x
+ + +
= = =
+ + +
atunci avem
1 2 3 2010
........ x x x x = = = = .
Prof. Nicolae Ivchescu, Craiova
G:436. Determinai cel mai mic numr natural n tiind c mprit la 108, 75, i 90 se obin resturile 57,
63 respectiv 48. Prof. Gheorghe Drstaru, Buzu
G:437. a) Calculai numrul
( )
3 1
2014 2014 1 2013 2015
161;
2014 2013 2015
n n
n
m
+ +
+
= +


b) S se afle numerele xyz scrise n baza zece tiind c
( )
7
,
m
a b c a c
e
+ +
N unde m este numrul
determinat la a). Prof. Iuliana Trac, Olt

G:438. Determinai o infinitate de cvadruple ) , , , ( d c b a care sunt soluii ale ecuaiei
5 4 3 2
d c b a = + + .
D.M. Btineu-Giurgiu, Bucureti i Neculai Stanciu, Buzu
G:439. Calculai ctul mpririi numrului 1 , 01 4 , 05 7 , 09 ... 43 , 57 a i i i i = + + + + la 3.
Prof. Sorina Vcrean, Cluj-Napoca
G:440. Fie A produsul primelor 100 de numere naturale nenule. Determinai numrul B
2
astfel nct B
2
s
fie maxim, s divid numrul A i s nu fie divizibil cu 6.
Prof. Petre Punescu, Roiorii de Vede
G:441. n ptratul ABCD, se consider punctele E pe (AB) i F pe (BC), astfel nct,
AE BF 1
EB FC 2
= = . Artai c

0
m(ADE) m(BEF) 45 + = .
Prof. Constantin Apostol, Rm. Srat
G:442. Artai c dac msurile unghiurilor unui triunghi sunt direct proporionale cu trei numere
naturale cu proprietatea c unul dintre ele este egal cu media aritmetic a celorlalte dou , atunci triunghiul
are un unghi cu msura de 60
0
. Prof. Simion Marin, Rm. Srat


Clasa a VII-a

G:443. Gsii xeNastfel ca
2
2 2012 x x + + s fie ptrat perfect.
Prof. Nicolae Ivchescu, Craiova
G:444. Artai c 2009 2010 2 4015 4, . x x x x + + > e1
Prof. Doina i Mircea Mario Stoica, Arad
G:445. Dac a, b i c sunt numere reale pozitive s se arate inegalitatea :
) ac bc ab ( 2 2 ) c b a (
2
+ + > + + .
Prof. erban George-Florin, Brila



- PROBLEME PROPUSE -

G:446. Fie
2
25
1
A x
x


= e e
`
+

)
1 N . Determinai probabilitatea ca alegnd un numr din
mulimea A, acesta s fie iraional i strict negativ. Prof. Sorina Vcrean, Cluj-Napoca
G:447. Folosind formule de calcul prescurtat, descompuneti n factori numrul 420 A = .
Prof. Petre Punescu, Roiorii de Vede
G:448. n triunghiul ABC,
| |
AM este median, iar ( ) P AB e astfel nct
1
.
2
AP
PB
= Dac
{ } AM CP N = ,artai c 4
ABC CMN
A A = . Prof . Gheorghe Drstaru, Buzu
G:449. Se consider dreptunghiul ABCD cu aria a (u.a.) i punctele M , respectiv N cu
) ( ), ( BC N AB M e e astfel nct aria triunghiului MND este b (u.a). Determinai minimul sumei
CN AM + n cazul n care b a 2 > .
D.M. Btineu-Giurgiu, Bucureti i Neculai Stanciu, Buzu
G:450. Determinai msurile unghiurilor triunghiului ABC, n care bisectoarea (BD) este
ct (DC) i BA AD BC + = . Prof. Constantin Apostol,Rm. Srat
G:451. S se arate c nu exist numere naturale nenule i distincte pentru care suma inverselor cuburilor
lor s fie egal cu
4
5
. Prof. Gheorghe Ghi, Buzu
G:452. Artai c n orice trapez isoscel ortodiagonal (cu diagonalele perpendiculare) , avem relaia :
h =
2
2
d
, unde h este lungimea nlimii iar d lungimea diagonalei trapezului.
Prof. Marin Simion, Rm. Srat




Clasa a VIII-a

G:453. Dac numerele reale nenule a,b, c, d verific relaiile 16 a b c d + + + = i
96 ab ac ad bc bd cd + + + + + = , s se arate c
1 1 1 1
a b c d
+ + + este numr natural.
Prof. Ionel Tudor, Clugreni, Giurgiu
G:454. Fie , a be1astfel nct ( 6) 2 ( 4) 17 0 a a b b + + + = . S se calculeze
2015 2015
2 ( ) ( ) a b b a + .
Prof. Adrian Stan,Buzu
G:455. Rezolvai n 1ecuaia
2 2 2
1 1 1 3
4 12 32 20 96 13 x x x x x x
+ + =
+ + + + +
.
Prof. Gheorghe Drstaru, Buzu
G:456. Fie funcia : , ( ) 2010. f f x x = + 1 1 Determinai funcia : , ( ( )) 7 4020 g f g x x = + 1 1 .
Prof. Mircea Mario Stoica, Arad
G:457. Dac y x, i z sunt numere reale nenule astfel nct
z y x z y x + +
= + +
1 1 1 1
atunci artai
c: ( )( ) = + + + +
2013 2011 2011 2011
z y x z y x ( )( )
2011 2013 2013 2013
z y x z y x + + + + .
D.M. Btineu-Giurgiu, Bucureti i Neculai Stanciu, Buzu
G:458. S se rezolve ecuaia:
2 4 6 2014
... 1.
2016 2016 2016 2016
x x x x ( ( ( (
+ + + + =
( ( ( (

unde | | x reprezint
partea ntreag a numrului x. Prof. Gheorghe Ghi, Buzu

PROBLEME PROPUSE


G:459. Artai c
_
_
n
n
9 ... 99 2
9 ... 99 26
3
eN , unde * neN . Prof. erban George-Florin, Brila
G:460. Pe perpendiculara n A, pe planul dreptunghiului ABCD, se consider punctul M. tiind c
laturile triunghiului MBD sunt proporionale cu numerele 34, 25 i 41 i c MC 15 2 = cm,
calculai dimensiunile dreptunghiului i lungimea segmentului (MA).
Prof. Constantin Apostol, Rm. Srat
G:461. Pe planul triunghiului ABC avnd m(

A) = 90
0
, AB = 6 cm i m(

C ) = 30
0
se ridic
perpendiculara AM. S se determine lungimea acestei perpendiculare tiind c msura unghiului diedru
format de planele (MBC) i (ABC) este de 60
0
. Prof. Marin Simion, Rm. Srat


Clasa a IX-a

L:307. Fie : f 1 1,
2
2
( 1) (2 1) 1
( )
2 1
m x m x m
f x
x mx m
+ + +
=
+ + +
. S se determine me1astfel nct
( ) 0 f x > pentru orice x numr real. Prof. Adrian Stan, Buzu
L:308. Dac n triunghiul ABC avem m(A) s m(B) s m(C) i a
2
+ b
2
= 2Rc s se calculeze m(C).
Prof. Marcel Chiri, Bucureti
L:309. Artai c n orice triunghi ABC , are loc inegalitatea p
a p
a
4
2
>

_
.
Prof. dr. Mihly Bencze, Braov
L:310. Dac: 8 7 , 7 6 , 6 5 , 5 4 , 4 3 , 3 2 = = = = = =
f e d c b a
, ct este valoarea produsului abcdef ?
D.M. Btineu-Giurgiu, Bucureti i Neculai Stanciu, Buzu
L:311. S se arate c n orice triunghi ABC, are loc inegalitatea: ,
2
) 4 ( 3
2
cos
2
cos
2
cos
R
R r C B A +
s + +
cu notaiile cunoscute. (In legtur cu problema C;2742, GM nr. 4/2004,Dinu Teodoresu, Pucioasa)
Prof. Gheorghe Ghi, Buzu

L:312. n interiorul triunghiului C AB se consider un punct K . Artai c dac:
2
2 sin
) (
2
A R
BKC A = A i
2
2 sin
) (
2
B R
CKA A = A , atunci K aparine mediatoarei segmentului
| |
AB .
Prof. Constantin Rusu, Rm. Srat

L:313. S se arate c exist x e1, pentru care
0 0
0
cos 25 sin 25
cos35
x +
eN .
Prof. Ionel Tudor, Clugreni, Giurgiu
L:314. Fie triunghiul C AB cu
0
( 90 ) m A = < i , ,
a a a
l m h sunt bisectoarea , mediana respectiv
nlimea corespunzatoare ipotenuzei . Dac
a a a
h m l >
2
atunci ABC A este dreptunghic isoscel .
Prof. erban George-Florin , Brila


- PROBLEME PROPUSE -


Clasa a X-a
L:315. S se arate c dac a
1
, a
2
, a
3
, a
4
sunt termenii unei progresii aritmetice cu termenii strict pozitivi
i cu raia pozitiv atunci :
n n n n
a a a a
3 2 4 1
1 1 1 1
+ < + , oricare ar fi neN, n > 2.
Prof. Chiri Marcel, Bucureti
L:316. Pentru *, peN se consider suma
3
3
0 0 0
( 1)( 2)( 3)
( )
( 1)( 2)( 3)
p m n
k
n
m n k
k k k
S p C
n n n
+
+
= = =
+ + +
=
+ + +
___
. S se rezolve
ecuaia ( ) 2 15. S p p + =
Prof. Ionel Tudor, Prof. Stelian Pican, Clugreni, Giurgiu
L:317. Dac irul ( )
0 > n n
a este definit prin 1
0
= a , 0
503
= a i pentru orice 1 > n , termenii
1 n
a ,
n
a a
1
i
1 + n
a sunt n progresie aritmetic, atunci determinai
2012
a .
D.M. Btineu-Giurgiu, Bucureti i Neculai Stanciu, Buzu
L:318. Fie ) , 0 ( ) 1 , ( , , + e c b a astfel nct
2
t
= + + arctgc arctgb arctga .
S se demonstreze c : ) 1 )( 1 )( 1 ( 3 3 8
2 2 2
c b a abc > .
Prof. Constantin Rusu, Rm. Srat
L:319. Dac , 3 , *, , , 1 , 0 ,
___
> e e = > m N m N n n i x
i i
o atunci are loc inegalitatea:
( )
.
...
) ... (
...
1
1 1
2
1
1
2 1
2
2
1
1


+ + +
+ + +
> + + +
m
m
n
m m
m
n
n
m
n
m m
x x x x x x
o o o
o o o

Prof. Gheorghe Ghi, Buzu
L:320. S se arate c 3 5 5 15 2 4 10, . n n n n + + + s + eN
Prof. Adrian Stan, Buzu
L:321. Fie , a b
+
e1 cu suma 1. Artai c
2
4
27
a b s , preciznd cnd are loc egalitatea.
Prof. Constantin Dinu, Buzu

Clasa a XI-a

L:322. Determinai matricile ) ( , Z M B A
n
e pentru care AB B A = + .
D.M. Btineu-Giurgiu, Bucureti i Neculai Stanciu, Buzu
L:323. S se studieze convergena irului :
1 ) 1 (
1 ) 1 (
2
2
+ +
+
=
o
n n
n
a
n
n
n
, R e o .
Prof. Constantin Rusu, Rm. Srat
L:324. Fie irul de numere reale
1
( )
n n
x
>
cu proprietatea 0
n
x ) i
1
1 2 3
( 2)
.... , 1.
4
n n
n
x x x
x x x x n
+
+ + + + = >
a) S se determine
1 2 3
, , ,.....
n
x x x x ;
b) S se arate c
1 2 3
.... 1
n
x x x x + + + + + eN. Prof. Adrian Stan, Buzu
L:325. ntr-un sistem de coordonate XOY se consider punctele (2;3), ( 3; 2), (0; ), A B C c c e1. S se
gseasc poziia punctului C astfel nct suma AC + BC s fie minim.
Prof. Constantin Dinu, Buzu


PROBLEME PROPUSE

L:326. Dac AeM
2
(R) i det(A + I
2
) = det(A
2
+ I
2
) atunci det(A) i Tr(A) iau valori n intervale de aceeai
lungime, unde det(A) i Tr(A) sunt determinantul, respectiv, urma matricei A.
Prof. Gheorghe Ghi, Buzu
L:327. Dac , ( )
n
A B M e C , inversabile i care verific relaia A B AB + =
atunci
( )
1
1 1
n
A B I


+ = . Prof. Marcel Chiri, Bucureti

L:328. Fie
2
( ) A M e o matrice astfel nct
2
2
A O = . Artai c
2
( ) B M e , numrul
det( ) N A AB BA = + + este raional.
Prof. Florin Stnescu, Geti, Dmbovia


Clasa a XII-a

L:329. Se consider funcia
4 3
: , ( ) 4 8 5 f f x x x x = + + 1 1 .
a) S se rezolve ecuaia f(x)=0;
b) Artai c exist ae1 pentru care (1 ) (1 ) 1; f a f a + = =
Prof. Ionel Tudor, Clugreni, Giurgiu
L:330. Calculai:
)

1
0
2014 2013
) 1 ( dx x x .
D.M. Btineu-Giurgiu, Bucureti i Neculai Stanciu, Buzu

L:331. Se consider o funcie ) , 0 [ ] , 0 [ : a f , derivabil cu derivata continu. S se arate c exist
] , 0 [ a c e astfel nct s avem:
|
.
|

\
|
'
>
)
2
) (
) ( ) (
0
c f a
a f a dx x f
a
.
Prof. Constantin Rusu, Rmnicu Srat
L:332. S se calculeze integrala:
2015 cos 2013
,
cos cos 2014
tg x x
dx
x x
)
pe un interval I pe care are sens funcia de
integrat.
Prof. Gheorghe Ghi, Buzu

L:333. tiind c 0 a) i 0 c( , stabilii care este numrul de rdcini reale pozitive ale polinomului
| |
3 2
P X aX bX c X = + + + e1 . Prof. Constantin Dinu, Buzu

L:334. Fie
| |
: ; * f a b
+
1 o funcie derivabil de dou ori, cu ''( ) f x continu i pozitiv pe
| |
; a b .
Dac f este strict cresctoare, artai c:


''( ) ( ) ( ) '( ) '( ) ( ) ( ) '( ) '( )
2 min ,
( ) ( ) '( ) ( ) ( ) ( ) '( )
b
a
f x f b f a f b f a f b f a f b f a
dx
f x f a f a f b f a f b f a

s + +
`

)
)
.

Prof. Florin Stnescu, Geti, Dmbovia
L:335. S se determine un polinom P(X) cu coeficieni ntregi astfel nct 3
n
+ P(n) s fie divizibil cu 32
pentru orice neN-.
Prof. Marcel Chiri, Bucureti



- QUICKIES-


5. QUICKIES
A Quickie should have an unexpected, succinct solution. Submitted quickies should not be under
consideration for publication elsewhere. We invite readers to submit solutions-quickies and new proposals-
quickies, accompanied by solutions mailed electronically (ideally MS Word 2003 or PDF file) to
stanciuneculai@yahoo.com. All communications should include the readers name, full address, and an e-mail
address. Submited solutions should arrive before September 30, 2014.

PROPOSALS - QUICKIES

Q9. Proposed by Nela Ciceu, Roiori, Bacu, Romania.
Let C B A , , be three collinear points and M be any point in the plane of the points C B A , , . The symmedian
from A in triangle ABM intersect MB in the point P , and the symmedian from A in
triangle ACM intersect MC in the point Q. Prove that the line PQ passes through a fixed point.
Q10. Proposed by D.M. Btineu-Giurgiu, and Neculai Stanciu, Romania.
If
*
, , ,
+
eR y x b a and
+
eR m , then prove that
m m
m
m
m
b a
y x
bx ay
y
by ax
x
) ( ) ( ) (
2 2 2 2
+
+
>
+
+
+
+ +
.
Q11. Proposed by Titu Zvonaru, Comneti, Romania.
Prove that if b a, and c are the lenghts of the sides of a triangle, then
3 2
2 2 2
>
(
(

|
.
|

\
|
+

+
|
.
|

\
|
+

+
|
.
|

\
|
+

+
+
+
+
+ a c
a c
c b
c b
b a
b a
c b a
c
b a c
b
a c b
a
.
Q12. Proposed by Mihly Bencze, Braov, Romania.
Prove that in all acute triangle ABC , holds
( )( ) ( )( )
_ _ _ _
> > A A ctgA tgA cos sin 4 3 9 3 .


SOLUTIONS QUICKIES

Q5. Proposed by D.M. Btineu-Giurgiu, and Neculai Stanciu, Romania.
Show that: if | ) e , 0 m , ( ) e , 0 , , , t z y x , then in any triangle ABC , with usual notations holds the
inequality
( )
( )
S
t z
y x
th zh
ym xa
m
m
m
cyclic
m
a c
m
b
) ( 3
) 3 4 (
2
1
1
2 2
1
2 2
+
+
>
+
+

+
+
_
.
Solution of Q5 by authors. By
c c b b a a
m h m h m h s s s , , , J. Radons inequality and
_ _
=
cyclic cyclic
a
a m
2 2
4
3
we
obtain
( )
( )
>
+
+
_
+
cyclic
m
a c
m
b
th zh
ym xa
2 2
1
2 2
( )
( )
=
|
|
.
|

\
|
+
|
|
.
|

\
|
+
>
+
+
_
_
_
+
+
m
cycclic
a c
m
cyclic
b
RADON
cyclic
m
a c
m
b
tm zm
ym xa
tm zm
ym xa
) (
) (
2 2
1
2 2
2 2
1
2 2

=
|
|
.
|

\
|
+
|
|
.
|

\
|
+
=
_
_ _
+
m
cycclic
a
m
m
cyclic
b
cyclic
m t z
m y a x
2
1
2 2
) (
=
|
|
.
|

\
|
+
|
.
|

\
|
|
|
.
|

\
|
+
_
_ _
+
m
cycclic
m
m
m
cyclic cyclic
a t z
a
y
a x
2
1
2 2
) (
4
3
4
3
=
|
|
.
|

\
|
+
|
.
|

\
|
|
|
.
|

\
|
+
_
_
+
+
+
m
cycclic
m
m
m
m
cyclic
m
a t z
a y x
2 1
1
2 1
) (
4
3
4
) 3 4 (


QUICKIES

m m
cyclic
m
t z
a y x
) ( 3 4
) 3 4 (
2 1
+
|
|
.
|

\
|
+
_
+
. By Ionescu-Weitzenbck inequality, i.e. S c b a 3 4
2 2 2
> + + and from above we
obtain
( )
( )
S
t z
y x
th zh
ym xa
m
m
m
cyclic
m
a c
m
b
) ( 3
) 3 4 (
2
1
1
2 2
1
2 2
+
+
>
+
+

+
+
_
, q.e.d.
Also solved by Marius Drgan, Bucharest, Romania.

Q6. Proposed by Titu Zvonaru, Comneti, and Neculai Stanciu, Romania.
How many digit has the number
96
2 ?
Solution of Q6 by authors. Using the inequality
3 10
10 1000 1024 2 = > = we obtain that:
( )
28 27
9
10 6 96
10 10 64 2 2 2 > > = .
We have succesively:
17 2 5 1700 5 1681 5 2 41 5 1025 1024 2
2 6 4 4 20 2 10
= < < = < =
29 2 5 290 5 289 5 2 17 5 2
13 12 12 36 6 18
= < < <
34 5 850 5 841 5 2 29 5 2
28 26 26 70 13 35
= < < <

29 67 2 29 28 28 69
5 2 2 5 20 5 17 5 2 < = < <
and then
29 96 29 29 29 67
10 2 2 5 2 2 < < .
From
29 96 28
10 2 10 < < we deduce that
96
2 has 28 digits.
Also solved by Marius Drgan, Bucharest, Romania.
Q7. Proposed by Mihly Bencze, Braov, Romania.
Solve in positive real numbers the equation

2 4 2 2 2
2
2
2 ) 2 4 (
4 4
log
+
+ + = + +
|
.
|

\
|
+
x x
x x
x
x
x
x .
Solution of Q7 by Titu Zvonaru and Daniel Vcaru, Romania.
Using the function
2
2 ) ( t t f
t
+ = , 0 > t , the given equation becomes
) 2 4 (
4
log
2
2
+ =
|
|
.
|

\
|
|
.
|

\
|
+ x x f
x
x f and because the function f is increasing ( 0 ) ( > ' t f for any 0 > t ),
so f is injective we obtain 2 4
4
log
2
2
+ = |
.
|

\
|
+ x x
x
x . We have 4
4
> +
x
x and
0 ) 2 ( 2 2 4
2 2
> s + x x x . Hence, we find one solution, and this is 2 = x .
Also solved by author.
Q8. Proposed by Babis Stergiou, Greece
Find two even and continuous functions R R g f : , with ) 1 ( ) 1 ( g f = and in addition with the
properties: ) ( ) ( 3
0
x xg dt t f
x
=
)
and ) ( ) ( 3
0
x xf dt t g
x
=
)
for all R x e .
Solution of Q8 by Daniel Vcaru, Piteti, Romania
Consider two primitives R R F : , R R G : such that ) ( ) ( x f x F = ' , ) ( ) ( x g x G = ' ,
R x e and 0 ) 0 ( ) 0 ( = = G F . So, ) ( ) ( 3 x xg x F = , ) ( ) ( 3 x xf x G = , R x e . Therefore we get
) ( ) ( 3 ) ( ) ( 3 x g x G x f x F = and multiplying by
3
2
we deduce that ) ( ) ( ) ( ) (
2 2
x G x F ' = ' , so
k x G x F = ) ( ) (
2 2
, R x e . Then 0 )) ( ) ( ))( ( ) ( ( = + x G x F x G x F , which yields
2
) ( ) ( ) ( ) ( 3 ) ( ) ( ) ( ) ( x x g x f x xf x F x g x f x G x F = = = = = , R x e
Also solved by Marius Drgan, Bucharest and Babis Stergiou, Greece.


- CALEIDOSCOP MATEMATIC -



Nimic nu este mai agil ca gndul ,
el te poart prin ntregul univers.
Thales

6. Caleidoscop matematic
1. Aranjeaz cele patru fructe n toate modurile posibile. Cte astfel de moduri exist ?






2. Dou borcane cu gem de caise de aceeai calitate
stau pe raftul unui magazin. Borcanul mai nalt este
cu dou treimi mai nalt dect cellalt i are
diametrul jumtate din diametrul borcanului mai
scurt. Dac borcanul mai nalt cost 5 lei iar cel mai
scurt, 9 lei, care cumprtur este mai avantajoas ?


3. Din vrful A al ptratului ABCD de latur 16 m,
pleac simultan un melc i o furnic. Dac viteza melcului este de 2,5 cm/s i a furnicii de 320 mm/s, s
se gseasc ce distan va fi ntre cei doi dup 3 minute.

4. Am mai multe pixuri care scriu colorat: 3 cu rou, 4 cu albastru, 5 cu verde i 2 cu negru.
n cte moduri pot scrie o scrisoare doar cu dou culori.

5. ntr-o grdin zoologic triesc 4 lei, 5 tigri i 6 pantere. Dac vrem s alegem un leu, un
tigru i dou pantere, n cte moduri o putem face ?
6. Dou cisterne au capacitatea de 8 m
3
,
respectiv 10 m
3
i sunt ncrcate cu 5 m
3
,
respectiv 6 m
3
de benzin pentru a fi
transportate la o staie. S se determine
care cistern este mai bine folosit n
raport cu capacitatea sa.

7. Cinci brbai i las la garderob
plriile care seamn ntre ele ns doamna de la garderob nu le aranjeaz n ordinea n care
le-a primit. Care este probabilitatea ca fiecare dintre cei cinci s-i primeasc propria plrie ?

8. Avem un dulap cu opt sertare ncuiate iar cheile nu sunt numerotate ca s tim crui sertar i
corespunde fiecare cheie. Cte ncercri trebuie s facem pentru a deschide toate sertarele?

9. ntr-un ifonier se afl 18 mnui: 4 perechi negre, 3 perechi verzi i 2 roii. Pe ntuneric, cte mnui
trebuie s alegi ca s fii sigur c printre acestea se afl cel puin dou de aceeai culoare ?
Rspunsuri: 1) 24; 2) borcanul mai scurt; 3) 1,9 m; 4) 6 situaii; 5) 300 moduri; 6) prima cistern;
7)
1
120
P = ; 8) 36 ncercri; 9) 4 mnui; (Rezolvrile detaliate vor fi puse n numrul urmtor.)

POSTA REDACTIEI


coala va fi coal cnd omul va fi om i statul va fi stat .
Mihai Eminescu






7. Pota redaciei

Dragi cititori, elevi i profesori, a aprut numrul 13 al revistei de matematic SCLIPIREA
MINTII, o revist care promoveaz studiul matematicii n rndul elevilor notri, i care, sperm noi, va
aduna tot mai muli elevi i profesori mpreun, din judeul Buzu i nu numai, pentru a face din obiectul
matematicii o activitate performant.
Profesorii i elevii care doresc s trimit materiale pentru revist, constnd n articole, exerciii i
probleme cu enun i rezolvare complet, materiale pentru caleidoscop matematic, sau orice alte sugestii
pentru a mbuntii calitatea acestei reviste, o pot face trimind materialele membrilor colectivului de
redacie sau pe adresa de e_mail: ady_stan2005@yahoo.com, fie materiale tehnoredactate( salvate n
Word 2003) , fie scrise de mn i scanate. Materialele primite trebuie s fie originale i s nu mai fi
fost trimise sau s mai fie trimise i ctre alte reviste. Dreptul de autor al materialelor trimise spre
publicare, aparine redaciei.
Data final pn cnd profesorii pot trimite materialele, rezolvrile i comenzile pentru numrul 14 al
revistei SCLIPIREA MINTII va fi 1 Octombrie 2014. V urm succes i v ateptm.


Rubrica rezolvitorilor de probleme

coala cu clasele I-VIII, Smrdan, Brdeanu
Clasa V-a: 15p. Anghel Mihaela; 9p. Neagu Alina, Furtun Aurelian, Grigore Denisa, Zoican Bogdan;
Clasa a VI-a: 19p. erban Larisa, Drugea Elena, Florea Mdlina, Andrei Florentina, Oprea Elisa.
Clasa a VII-a: 20p. Furtun Elena, Neagu Andra, Stanciu Catrinel, Ilie Adina. Prof. Stanescu Ion.
coala cu clasele I-VIII, Tristan Tzara Moineti, Bacu
Clasa a V-a : 18p. Paduraru Catalin, Dobrovat Alina, Munteanu Diana, Ailincai iulian, Ciubotariu Costin.
Clasa a VI-a: 45p.Maziliu Alexandra, Gherman Andrada, Faraoneanu Madalina.
Clasa a VII-a : 55p. Munteanu Gheorghe, Lila Ionut. Prof. Cornelia Guru;
coala cu clasele I-VIII, nr. 2 Cugir, Alba
Clasa a V-a: 25p. Bel Luana;
Clasa a VII-a: 65p. Codrea Maria, Codrea Marian, Muntean Ioan, Molode Drago, Sideria Alexandru;
Prof. Mariana Mitea.
coala cu clasele I-VIII Gh. Popescu Mrgineni Slobozia, Olt
Clasa a VII-a: 53p. Trac Ionu- Vldu, Ene Daniela- Iuliana, Vochin Ioan David, Moisescu Denis,
Costea Nicoleta, Nedelea Adrian Ilie, Mmularu Cristina, Fuior Daniela; Prof. Iuliana Trac.
Liceul Tehnologic Costin Neniescu, Buzu
Clasa a X-a :35p. Stroe Ionu, Bljanu Ioana, Dogaru Ana Maria, Croitoru Andrei.
Clasa a XI-a: 59p. Tureac Andrei, Stoian Cristina, Sterpu Denisa, Sandu Cristina, Pirnog Georgiana,
Ciopec Corina, 53p. Dobrin Iulian, Dragomir Ionu , Picu Elena Daniela,; Prof. Stan Adrian.
coala erban Cioculescu, Geti, Dmbovia.Clasa a VII-a: 10p. Maierean Alex, Petre Adriana,
Tudor Georgiana, Mihalache Alexandra, Badea Valeria;
Clasa a VIII-a: 15p. Pun Alexandra, Bucur Diana, Serafim Laureniu; Prof. Florin Stnescu.

S-ar putea să vă placă și